Mental Health Exam Two Study Questions

Lakukan tugas rumah & ujian kamu dengan baik sekarang menggunakan Quizwiz!

Pam, the nurse educator, is teaching a new nurse about seclusion and restraint. Order the following interventions from least (1) to most (5) restrictive: a. With the patient identify the behaviors that are unacceptable and consequences associated with harmful behaviors b. Placing the patient in physical restraints c. Allowing the patient to take a time-out and sit in his or her room d. Offering a PRN medication by mouth e. Placing the patient in a locked seclusion room

1-a. With the patient identify the behaviors that are unacceptable and consequences associated with harmful behaviors 2-d. Offering a PRN medication by mouth 3-c. Allowing the patient to take a time-out and sit in his or her room 4-e. Placing the patient in a locked seclusion room 5-b. Placing the patient in physical restraints

What behaviors might a patient experiencing a panic level of anxiety demonstrate?

Behaviors that may be manifested include: pacing running shouting screaming Physical behavior may become: chaotic uncoordinated impulsive

A child has been traumatized by a history of violence. The nurse plans to assess the child's mental health. What are some things that should be assessed?

Broad categories to assess include safety, general appearance, socialization, activity level, speech, coordination and motor function, affect, manner of relating, intellectual function, thought processes and content, and characteristics of play. Symptoms such as uncontrollable rage, somatic symptoms, posttraumatic symptoms (e.g., nightmares, night terrors, disturbing hallucinations, intrusive traumatic thoughts and memories, re-experiencing or flashbacks, traumatic re-enactments, and self-injurious behaviors), as well as negative symptoms (e.g., numbing and avoidance). Somatic symptoms may manifest as headaches, stomach aches, or pain; memory problems include amnesia, forgetfulness, difficulty concentrating, and trance states. Comorbid conditions should be assessed. Developmental testing should be done.

Linda is terrified of spiders and cannot explain why. Because she lives in a wooded area, she would like to overcome the overwhelming fear. Her nurse practitioner suggests which kind of therapy? A-behavioral B-biofeedback C-aversion D-systemic desensitization

D-systemic desensitization

A patient was admitted to your unit with Bipolar I disorder and is in the manic state. What symptoms do you expect to see

Extreme drive and energy Inflated sense of self-importance Decreased sleep requirements Excessive talking and pressured speech Personal feeling of racing thoughts, distraction by environmental events Usually obsessed and overfocused goals Purposeless arousal and movement Dangerous activities - indiscriminate spending, risky investments, reckless sexual encounters

What are some familial risk factors that appear to be related to a child's experiencing a psychiatric disorder?

Familial risk factors that correlate with psych disorders in children include: 1-severe marital discord 2-low scoioeconomic status 3-large families and overcrowding 4-parental criminality 5-maternal psych disorders 6-foster care placement Other factors for mental health problems: 1-witnessing violence 2-children who have experienced abuse 3-bullying

What would be some appropriate interventions for the parent whose child is missing at the mall and is experiencing panic level anxiety?

Maintain a calm manner Remain with the patient Minimize environmental stimuli if possible Move to a quieter setting if possible and stay with the patient Use clear, simple sentences and repetition Use a low-pitched voice and speak calmly Recognize person is anxious and in distress Be willing to listen

A 15 year old is admitted to your unit with anorexia. You have completed your physical assessment of her. What are some important biopsychosocial areas to assess?

Patients perception of the problem Eating habits History of dieting Methods used to achieve weight control (restricting, purging, exercising) Value attached to a specific shape and weight Interpersonal and social functioning Mental status and physiological parameters

You believe a young man you are admitting to your unit is suffering from command hallucinations. What would be some questions to ask him? After acute admission, discharge is being planned for this patient. What are some things that need be be considered?

Questions to include: 1.do you recognize the voice? 2.Do you believe the voices are real? 3.Do you plan to follow the command? Things to be considered include: 1.External factors - patient's living arrangement, economic sources, social supports, family relationships 2.Internal factors - resilience, range of coping skills 3."connecting patient and family" with community resources that provide therapeutic programing and social, financial, and other needed support

What are some common problems that can be avoided if a manic patient gets proper treatment?

Suicide attempts Alcohol or substance abuse Marital or work problems Developmental of medical comorbidity

What are some interventions the nurse could use with this child who has been traumatized?

Teach deep breathing techniques. Soothing strategies might also include warm baths, singing, redirecting, distraction, listening to music, guided imagery, and using a low, calming voice.

A new nurse on a psychiatric unit wants to effectively communicate in a therapeutic relationship with a patient. What are some things that will help this communication?

The nurse should: 1-know what they are trying to convey (purpose of the message) 2-communicate what is really meant to the patient 3-comprehend the meaning of what the patient is intentionally/unintentionally conveying Communication should include: 1-clarify: meaning or message accurately understood by both parties 2-continuity: promotes connections among ideas

Your patient has been started on lithium. What patient teaching about this medication should the nurse provide before the patient is discharged?

The patient should be given careful instructions about: 1-purpose and requirement of lithium therapy 2-adverse effects 3-toxic effects and complications 4-when to contact doctor 5-need for periodic blood tests Two long term risks include hypothyroidism and impairment of kidneys' ability to concentrate urine

Your patient was diagnosed with a major depressive disorder. What medication do you anticipate on the health care provider prescribing? What side effects might the patient experience?

The selective serotonin reuptake inhibitors (SSRIs) are recommended as first-line therapy for most types of depression. Examples are fluoxetine (Prozac), sertraline (Zoloft), paroxetine (Paxil), citalopram (Celexa), escitalopram (Lexapro), and fluvoxamine (Luvox). May induce agitation, anxiety, sleep disturbance, tremor, sexual dysfunction (primarily anorgasmia), or tension headache. The effect of the SSRIs on sexual performance may be the most significant undesirable outcome reported by patients. Autonomic reactions (e.g., dry mouth, sweating, weight change, mild nausea, and loose bowel movements) may also be experienced with the SSRIs.

If the child is diagnosed with autism, what type of treatment will be reccomended?

Treatment plans: 1-behavior management with reward system that teaches parents to provide structure and rewards, consistency in rules and expectations at home in order to shape and modify behavior and foster development of socially appropriate skills 2-child may receive PT, OT, and speech therapy as part of care team

Your best friend tells you that she is experiencing insomnia. What are some questions you could ask her to help assess what might be going on?

When did you begin having trouble with sleep? Have you had trouble with sleep in the past? Describe your pre-bedtime routine. What are the activities you customarily engage in before sleep? Describe your sleeping environment. Are there things in your sleep environment that are hampering your sleep (e.g., noise, light, temperature, overall comfort)? Do you use your bedroom for things other than sleep or sexual activity (e.g., working, eating, or watching television)? What time do you go to bed? How long does it take to fall asleep? Once asleep, does middle-of-the-night awakening disturb you? If so, what wakes you up? Are you able to return to sleep? If you are unable to sleep, what do you do? What time do you wake up? What time do you get out of bed? How much time do you actually think you sleep? Do you sleep longer on weekends or days off? Do you nap? If so, for how long? Do you feel refreshed after napping? Can you identify any stress or problem that may have initially contributed to your sleep difficulties? Tell me about your daily habits, your diet, exercise, and medications. What changes, if any, have you made to improve your sleep? What were the results?

Pyromania, a behavior associated with impulse control disorders, causes an individual to engage in what behavior? a.Starting fires b.Stealing for thrill c.Self-mutilate d.Directing anger toward others

a.Starting fires

Which statement describes a common sexual side effect of diazepam (Valium)? a. "I'm just not interested in sex as much." b. "I'm experiencing vaginal dryness." c. "I don't have organisms anymore." d. "My breasts have gotten larger."

a. "I'm just not interested in sex as much."

Which patient statement supports the diagnosis of anorexia nervosa? a. "I'm terrified of gaining weight." b. "I wish I had a good friend to talk to." c. "I've been told I drink way too much alcohol." d. "I don't get much pleasure out of life anymore."

a. "I'm terrified of gaining weight."

Two months ago, Natasha's husband died suddenly and she has been overwhelmed with grief. When Natasha is subsequently diagnosed with major depressive disorder, her daughter, Nadia, makes which true statement? a. "Depression often begins after a major loss. Losing dad was a major loss." b. "Bereavement and depression are the same problem." c. "Mourning is pathological and not normal behavior." d. "Antidepressant medications will not help this type of depression."

a. "Depression often begins after a major loss. Losing dad was a major loss."

What assessment question should the nurse ask when attempting to determine a teenager's mental health resilience? Select all that apply. a. "How did you cope when your father deployed with the Army for a year in Iraq?" b. "Who did you go to for advice while your father was away for a year in Iraq?" c. "How do you feel about talking to a mental health counselor?" d. "Where do you see yourself in 10 years?" e. "Do you like the school you go to?"

a. "How did you cope when your father deployed with the Army for a year in Iraq?" b. "Who did you go to for advice while your father was away for a year in Iraq?" d. "Where do you see yourself in 10 years?"

Which statement made by the patient demonstrates an understanding of the treatment of choice for patients managing the effects of traumatic events? a. "I attend my therapy sessions regularly." b. "Those intrusive memories are hidden for a reason and should stay hidden." c. "Keeping busy is the key to getting mentally healthy." d. "I've agreed to move in with my parents so I'll get the support I need."

a. "I attend my therapy sessions regularly."

Which statement made by the patient demonstrates an understanding of the effective use of newly prescribed lithium to manage bipolar mania? Select all that apply. a. "I have to keep reminding myself to consistently drink six 12-ounce glasses of fluid every day." b. "I discussed the diuretic my cardiologist prescribed with my psychiatric care provider." c. "Lithium may help me lose the few extra pounds I tend to carry around." d. "I take my lithium on an empty stomach to help with absorption." e. "I've already made arrangements for my monthly lab work."

a. "I have to keep reminding myself to consistently drink six 12-ounce glasses of fluid every day." b. "I discussed the diuretic my cardiologist prescribed with my psychiatric care provider." e. "I've already made arrangements for my monthly lab work."

Taylor, a psychiatric registered nurse, orients Regina, a patient with anorexia nervosa, to the room where she will be assigned during her stay. After getting Regina settled, the nurse informs Regina: a. "I need to go through the belongings you have brought with you." b. "You can use the scale in the back room when you need to." c. "You will be eating five times a day here." d. "The daily structure is based around your desire to eat."

a. "I need to go through the belongings you have brought with you."

Which nursing response demonstrates accurate information that should be discussed with the female patient diagnosed with bipolar and her support system? Select all that apply. a. "Remember that alcohol and caffeine can trigger a relapse of your symptoms" b. "Due to the risk of a manic episode, antidepressant therapy is never used with bipolar disorder. c. "It's critical to let your healthcare provider know immediately if you aren't sleeping well." d. "Is your family prepared to be actively involved in helping manage this disorder?" e. "The symptoms tend to come and go and so you need to be able to recognize the early signs."

a. "Remember that alcohol and caffeine can trigger a relapse of your symptoms" c. "It's critical to let your healthcare provider know immediately if you aren't sleeping well." d. "Is your family prepared to be actively involved in helping manage this disorder?" e. "The symptoms tend to come and go and so you need to be able to recognize the early signs."

When considering an eating disorder, what is a physical criterion for hospital admission? a. A daytime heart rate of less than 50 beats per minute b. An oral temperature of 100°F or more c. 90% of ideal body weight d. Systolic blood pressure greater than 130 mm Hg

a. A daytime heart rate of less than 50 beats per minute

A young child is found wandering alone at a mall. A male store employee approaches and asks where her parents are. She responds, "I don't know. Maybe you will take me home with you?" This sort of response in children may be due to: a. A lack of bonding as an infant b. A healthy confidence in the child c. Adequate parental bonding d. Normal parenting

a. A lack of bonding as an infant

Sleep disturbances are often overlooked or undiagnosed due to: a. A lack of formal nurse and physician training in sleep disturbances b. Patients not often accurately describing sleep disturbance patterns c. The belief that sleep disturbance is a necessary part of hospitalization d. Patients hiding the fact that they have issues with sleep

a. A lack of formal nurse and physician training in sleep disturbances

Which patient has the greatest risk for suicide? a. A patient who expresses the inability to stop searching the internet for child pornography. b. A patient who reports having lost interest in having a sexual relationship with his wife. c. A patient with a history of exposing himself to female strangers on the bus. d. A patient whose attraction to prepubescent girls has increased.

a. A patient who expresses the inability to stop searching the internet for child pornography.

Malika has been overweight all of her life. Now an adult, she has health problems related to her excessive weight. Seeking weight loss assistance at a primary care facility Malika is surprised when the nurse practitioner suggests: a. A trial of SSRI antidepressant therapy b. Mild exercise to start, increasing in intensity over time c. Removing snack foods from the home d. Medication treatment for hypertension

a. A trial of SSRI antidepressant therapy

To provide effective care for the patient diagnosed with schizophrenia, the nurse should frequently assess for which associated condition? Select all that apply. a. Alcohol use disorder b. Major depressive disorder c. Stomach cancer d. Polydipsia e. Metabolic syndrome

a. Alcohol use disorder b. Major depressive disorder d. Polydipsia e. Metabolic syndrome

Which characteristic in an adolescent female is sometimes associated with the prodromal phase of schizophrenia? a. Always afraid another student will steal her belongings. b. An unusual interest in numbers and specific topics. c. Demonstrates no interest in athletics or organized sports. d. Appears more comfortable among males.

a. Always afraid another student will steal her belongings.

Which behaviors will the nurse encourage a patient diagnosed with insomnia disorder to adopt? Select all that apply. a. Avoiding exercising at bedtime b. Avoiding napping during the day c. Eating a hearty snack at bedtime d. Getting up at the same time each day e. Moving the clock so it is not visible from the bed

a. Avoiding exercising at bedtime b. Avoiding napping during the day d. Getting up at the same time each day e. Moving the clock so it is not visible from the bed

The activity of gamma-aminobutyric acid (GABA) contributes to a slowing of neural activity. Which of the following drugs facilitates the action of GABA? a. Benzodiazepines b. Antihistamines c. Anticonvulsants d. Noradrenergic

a. Benzodiazepines

In pediatric mental health there is a lack of sufficient numbers of community-based resources and providers, and there are long waiting lists for services. This has resulted in: Select all that apply. a. Children of color and poor economic conditions being underserved b. Increased stress in the family unit c. Markedly increased funding d. Premature termination of services

a. Children of color and poor economic conditions being underserved b. Increased stress in the family unit d. Premature termination of services

When discussing oppositional defiant disorder with a group of parents, what information should the nurse include about the disorder? Select all that apply a. Classic symptoms include anger, irritation, and defiant behavior. b. Children generally outgrow the behaviors without formal treatment. c. Severity is considered mild when symptoms are present in only one setting. d. Disorder is diagnosed equally in both males and females. e. Argumentative and defiant are terms often used to describe the patient.

a. Classic symptoms include anger, irritation, and defiant behavior. b. Children generally outgrow the behaviors without formal treatment. c. Severity is considered mild when symptoms are present in only one setting. e. Argumentative and defiant are terms often used to describe the patient.

Which treatment is typically prescribed for primary insomnia? Select all that apply. a. Cognitive-behavioral therapy-insomnia (CBT-I) b. Intravenous medication for sedation c. Stimulus control d. Sleep restriction e. Sleep hygiene measures

a. Cognitive-behavioral therapy-insomnia (CBT-I) b. Intravenous medication for sedation c. Stimulus control d. Sleep restriction e. Sleep hygiene measures

Which nursing intervention focuses on managing a common characteristic of major depressive disorder associated with the older population? a. Conducting routine suicide screenings at a senior center. b. Identifying depression as a natural, but treatable result of aging. c. Identifying males as being at a greater risk for developing depression. d. Stressing that most individuals experience just a single episode of major depression in a lifetime.

a. Conducting routine suicide screenings at a senior center.

Melanie is a 38-year-old female admitted to the hospital to rule out a neurological disorder. The testing was negative, yet she is reluctant to be discharged. Today she has added lower back pain and a stabbing sensation in her abdomen. The nurse suspects a factitious disorder in which Melanie may: a. Consciously be trying to maintain her role of a sick patient b. Not recognize her unmet needs to be cared for c. Protect her child from illness d. Recognize physical symptoms as a coping mechanism

a. Consciously be trying to maintain her role of a sick patient

Tommy, a 12-year-old boy admitted to the pediatric psychiatric unit, has recently been diagnosed with conduct disorder. In the activity room, the games he wanted to play were already in use. He responded by threatening to throw furniture and to hurt his peers who had the game he wanted. Nancy, a registered nurse, recognizes that Tommy's therapy must include: a. Consistency in implementing the consequences of breaking rules b. Empathetic reasoning when Tommy acts out in the activity room c. Teaching Tommy the benefits of socializing d. Solitary time so that Tommy can think about his actions

a. Consistency in implementing the consequences of breaking rules

What assessment data would support a diagnosis of conduct disorder? Select all that apply. a. Evidence of social isolation b. Arrested twice for disorderly conduct c. Expresses difficulty in keeping employment d. Demonstrates objective signs of phobia e. Exhibits signs of chronic self-mutilation

a. Evidence of social isolation b. Arrested twice for disorderly conduct c. Expresses difficulty in keeping employment

Which patient has an increased risk for the development of anxiety and will require frequent assessment by the nurse? Select all that apply. a. Exacerbation of asthma signs and symptoms b. History of peanut and strawberry allergies c. History of chronic obstructive pulmonary disease d. Current treatment for unstable angina pectoris e. History of a traumatic brain injury

a. Exacerbation of asthma signs and symptoms c. History of chronic obstructive pulmonary disease d. Current treatment for unstable angina pectoris e. History of a traumatic brain injury

Tammy, a 28-year-old with major depressive disorder and bulimia nervosa, is ready for discharge from the county hospital after 2 weeks of inpatient therapy. Tammy is taking citalopram (Celexa) and reports that it has made her feel more hopeful. With a secondary diagnosis of bulimia nervosa, what is an alternative antidepressant to consider? a. Fluoxetine (Prozac) b. Isocarboxazid (Marplan) c. Amitriptyline d. Duloxetine (Cymbalta)

a. Fluoxetine (Prozac)

Safety measures are of concern in eating-disorder treatments. Patients with anorexia nervosa are supervised closely to monitor: Select all that apply. a. Foods that are eaten b. Attempts at self-induced vomiting c. Relationships with other patients d. Weight

a. Foods that are eaten b. Attempts at self-induced vomiting d. Weight

Kyle, a patient with schizophrenia, began to take the first-generation antipsychotic haloperidol (Haldol) last week. One day you find him sitting very stiffly and not moving. He is diaphoretic, and when you ask if he is okay he seems unable to respond verbally. His vital signs are: BP 170/100, P 110, T 104.2°F. What is the priority nursing intervention? Select all that apply. a. Hold his medication and contact his prescriber. b. Wipe him with a washcloth wet with cold water or alcohol. c. Administer a medication such as benztropine IM to correct this dystonic reaction. d. Reassure him that although there is no treatment for his tardive dyskinesia, it will pass. e. Hold his medication for now and consult his prescriber when he comes to the unit later today.

a. Hold his medication and contact his prescriber. b. Wipe him with a washcloth wet with cold water or alcohol.

A male arrested for inappropriate sexual contact in a subway car denies the allegation. Upon interviewing the man, the nurse suspects frotteuristic disorder due to his: a. Lack of relationships b. Overall aggressive nature c. Criminal history including robbery d. Intense hatred of women

a. Lack of relationships

Which factors tend to increase the difficulty of diagnosing young children who demonstrate behaviors associated with mental illness? Select all that apply. a. Limited language skills b. Level of cognitive development c. Level of emotional development d. Parental denial that a problem exists e. Severity of the typical mental illnesses observed in young children

a. Limited language skills b. Level of cognitive development c. Level of emotional development

The stage of sleep known as rapid eye movement or REM sleep is characterized by atonia and myoclonic twitches in addition to the actual rapid movement of the eyes. Atonia is thought to be a protective mechanism as it: a. Limits physical movements b. Prevents nightmares c. Enhances the dream state d. Regulates the autonomic nervous system

a. Limits physical movements

The impulse control spectrum can begin in childhood and continue on into adulthood, often morphing into criminal behaviors. Working with patients diagnosed with these disorders, the best examples of expressed emotion by the nursing staff are: a. Low to prevent emotional reactions b. Matched to the patient's level of emotion c. Flat without evidence of any emotional output d. High expression to improve therapeutic patient emotions

a. Low to prevent emotional reactions

A pregnant woman is in a relationship with a male who routinely abuses her. Her unborn child may engage in high-risk behavior as a teen as a result of: a. Maternal stress b. Parental nurturing c. Appropriate stress responses in the brain d. Memories of the abuse

a. Maternal stress

Which intervention should the nurse implement when caring for a patient demonstrating manic behavior? Select all that apply. a. Monitor the patient's vital signs frequently. b. Keep the patient distracted with group-oriented activities. c. Provide the patient with frequent milkshakes and protein drinks. d. Reduce the volume on the television and dim bright lights in the environment. e. Use a firm but calm voice to give specific concise directions to the patient.

a. Monitor the patient's vital signs frequently. c. Provide the patient with frequent milkshakes and protein drinks. d. Reduce the volume on the television and dim bright lights in the environment. e. Use a firm but calm voice to give specific concise directions to the patient.

Which chronic medical condition is a common trigger for major depressive disorder? a. Pain b. Hypertension c. Hypothyroidism d. Crohn's disease

a. Pain

When considering the need for monitoring, which intervention should the nurse implement for a patient with anorexia nervosa? Select all that apply. a. Provide scheduled portion-controlled meals and snacks. b. Congratulate patients for weight gain and behaviors that promote weight gain. c. Limit time spent in bathroom during periods when not under direct supervision. d. Promote exercise as a method to increase appetite. e. Observe patient during and after meals/snacks to ensure that adequate intake is achieved and maintained.

a. Provide scheduled portion-controlled meals and snacks. c. Limit time spent in bathroom during periods when not under direct supervision. e. Observe patient during and after meals/snacks to ensure that adequate intake is achieved and maintained.

Which nursing intervention is particularly well chosen for addressing a population at high risk for developing schizophrenia? a. Screening a group of males between the ages of 15 and 25 for early symptoms. b. Forming a support group for females aged 25 to 35 who are diagnosed with substance use issues. c. Providing a group for patients between the ages of 45 and 55 with information on coping skills that have proven to be effective. d. Educating the parents of a group of developmentally delayed 5- to 6-year-olds on the importance of early intervention.

a. Screening a group of males between the ages of 15 and 25 for early symptoms.

What is a common behavior observed in a patient diagnosed with intermittent explosive disorder? Select all that apply. a. Short attention span b. Threatens suicide c. Often purges after eating d. Uses alcohol to excess e. States, "Everyone is out to get me."

a. Short attention span b. Threatens suicide d. Uses alcohol to excess

Which medications are currently approved for the treatment of male erectile disorder? Select all that apply. a. Sildenafil (Viagra) b. Flibanserin (Addyi) c. Tadalafil (Cialis) d. Vardenafil (Levitra) e. Avanafil (Stendra)

a. Sildenafil (Viagra) c. Tadalafil (Cialis) d. Vardenafil (Levitra) e. Avanafil (Stendra)

In a parent teacher conference, the school nurse meets with the parents of a profoundly shy 8-year-old girl. The parents hold hands, speak softly, respond briefly, and have poor eye contact. The nurse recognizes that the child is most likely exposed to parental modeling and: a. The inherited shyness trait b. A lack of affection in the home c. Severe punishment by the parents d. Is afraid to say something foolish

a. The inherited shyness trait

A child diagnosed with attention deficit hyperactivity disorder (ADHD) is reprimanded for taking the nurse's pen without asking first. He responds by shouting, "You don't like me! You won't let me have anything, even a pen!" The nurse is most therapeutic when responding with which statement? a."I do like you, but I don't like it when you grab my pen." b."Liking you has nothing to do with whether I will loan you my pen." c."It sounds as though you are feeling helpless and insecure." d."You must ask for permission before taking someone else's things."

a."I do like you, but I don't like it when you grab my pen." This reply shows positive regard for the child while describing the behavior as undesirable. Feedback such as this helps the child feel accepted while making her aware of the effect her behavior has on others.

Which statement by a patient who was educated about the importance of acquiring adequate sleep indicates a need for further teaching? a."I will be sure to try to get 8 hours of sleep every night, and 9 or 10 hours of sleep if I can." b."Getting less than 6 hours of sleep at night may increase my risk for medical problems." c."Getting enough sleep will increase my productivity at work." d."Since I have to drive for my job, getting enough sleep will help me avoid accidents."

a."I will be sure to try to get 8 hours of sleep every night, and 9 or 10 hours of sleep if I can." Sleeping more than 8 hours per night is associated with up to a twofold increased risk of obesity, diabetes, hypertension, incident cardiovascular disease, stroke, depression, and substance abuse. The other options are all true.

One criterion for the diagnosis of primary insomnia is met when the client makes which statement? a."I've actually missed work because I'm too tired to go." b."I was diagnosed with depression 2 months ago." c."I've had problems falling asleep for 3 weeks now." d."I have these terrible nightmares when I fall asleep."

a."I've actually missed work because I'm too tired to go." A criterion for primary insomnia listed in the DSM-5 is disruption of the normal routine as a result of the sleep disturbance.

A depressed, socially withdrawn client tells the nurse, "There is no sense in trying. I am never able to do anything right!" The nurse can best address this cognitive distortion with which response? a."Let's look at what you just said, that you can 'never do anything right.'" b."Tell me what things you think you are not able to do correctly." c."Is this part of the reason you think no one likes you?" d."That is the most unrealistic thing I have ever heard."

a."Let's look at what you just said, that you can 'never do anything right.'" Cognitive distortions can be refuted by examining them, but to examine them the nurse must gain the client's willingness to participate. None of the other options examines the underlying cause of the feeling.

When discussing the symptoms of post-traumatic stress disorder (PTSD), the nurse should make which statement? a."The symptoms can occur almost immediately or can take years to manifest." b."PTSD causes agitation and hypervigilance but rarely chronic depression." c."When experiencing a flashback, the client generally experiences a slowing of responses." d."PTSD is an emotional response that does not cause significant changes in brain chemistry."

a."The symptoms can occur almost immediately or can take years to manifest." The onset of PTSD symptoms can occur as early as a month after exposure, but a delay of months or years is not uncommon.

A client who is demonstrating a moderate level of anxiety tells the nurse, "I am so anxious, and I do not know what to do." Which response should the nurse make initially? a."What things have you done in the past that helped you feel more comfortable?" b."Let's try to focus on that adorable little granddaughter of yours." c."Why don't you sit down over there and work on that jigsaw puzzle?" d."Try not to think about the feelings and sensations you're experiencing."

a."What things have you done in the past that helped you feel more comfortable?" Because the client is not able to think through the problem and arrive at an action that would lower anxiety, the nurse can assist by asking what has worked in the past. Often what has been helpful in the past can be used again. While distraction may be helpful in some situations, it is not the initial intervention

Which child is demonstrating behaviors that support a diagnosis of adolescent onset conduct disorder? a.A 12-year-old male who steals a bicycle as a gang initiation b.A 9-year-old male who smokes half a pack of cigarettes a day c.A 12-year-old female who regularly bullies her younger siblings d.A 9-year-old female who engages in sexually provocative behaviors

a.A 12-year-old male who steals a bicycle as a gang initiation In adolescent-onset conduct disorder, no symptoms are present prior to age 10. Affected adolescents tend to act out misconduct with their peer group (e.g., early onset of sexual behavior, substance abuse, risk-taking behaviors). Males are more likely to fight, steal, vandalize, and have school discipline problems, whereas girls tend to lie, be truant, run away, abuse substances, and engage in prostitution

Currently what is understood to be the causation of schizophrenia? a.A combination of inherited and nongenetic factors b.Deficient amounts of the neurotransmitter dopamine c.Excessive amounts of the neurotransmitter serotonin d.Stress related and ineffective stress management skills

a.A combination of inherited and nongenetic factors Causation is a complicated matter. Schizophrenia most likely occurs as a result of a combination of inherited genetic factors and extreme nongenetic factors (e.g., viral infection, birth injuries, nutritional factors) that can affect the genes governing the brain or directly injure the brain

A 23 years old is admitted with reports of abdominal pain, dizziness, and headache. When told that all the results of a physical workup have been negative, the client shares, "Now I am having back pain." Which notation in the client's medical record may alert the nurse to the possibility of malingering? a.A court date this week for drunk driving b.Was adopted at the age of 5 years c.A history of physical abuse by his stepfather d.A history of oppositional-defiant disorder e.Raised primarily in a single parent household

a.A court date this week for drunk driving Malingering is a process of fabricating an illness or exaggerating symptoms to gain a desired benefit or avoid something undesired, such as to obtain prescription medications, evade military service, or evade legal action. It is more common in men, those who have been neglected or abused in childhood, and those who have had frequent childhood hospitalizations. Adoption is not known to be a causative factor in malingering. A history of oppositional-defiant disorder is not known to a causative factor in malingering. Being raised in a single parent home is not known to be a causative factor in malingering

What is the initial task of an outpatient clinic nurse who is working with a client experiencing a sexual disorder? a.Establish trust with the client b.Assess the client's physical health c.Explain that the nurse is a therapeutic agent d.Orient the client to the clinic's programs, use as part of therapy

a.Establish trust with the client The initial task in working with any client is to establish trust.

The nurse anticipates that the nursing history of a client diagnosed with obsessive compulsive disorder (OCD) will reveal what common assessment data? Select all that apply. a.A history of childhood traumab b.A sibling with the disorder c.A history of sexual abuse d.A previous suicide attempt e.An eating disorder

a.A history of childhood traumab b.A sibling with the disorder c.A history of sexual abuse e.An eating disorder Sexual and physical abuse in childhood or trauma increases the risk of this disorder. Genetics are strongly associated with this disorder. First-degree relatives have twice the risk. OCD tends to occur along with anxiety disorders 76% of the time. Other comorbid conditions include major depressive disorder, bipolar disorder, and eating disorders.

What is the most effective nursing intervention regarding the accurate assessment of sleep disorders? a.A sleep diary b.Information regarding sleep cycles c.Client description of the symptomatology d.Assessment for substance abuse

a.A sleep diary Self-reported sleep patterns may be biased, and so clinical tools such as a sleep diary are helpful in accurately estimating total sleep time.

The family of a child diagnosed with attention deficit hyperactivity disorder (ADHD), inattentive type, is told the evaluation of their child's care will focus on symptom patterns and severity. What is the focus of child's evaluation? Select all that apply. a.Academic performance b.Activities of daily living c.Physical growth d.Social relationships e.Personal perception

a.Academic performance b.Activities of daily living d.Social relationships e.Personal perception For the family and child with ADHD, evaluation will focus on the symptom patterns and severity. For those with ADHD, inattentive type, the focus of evaluation will be academic performance, activities of daily living, social relationships, and personal perception. For those with ADHD, hyperactive-impulsive type or combined type, the focus will be on both academic and behavioral responses.

Which statement about structural dissociation of the personality is true? a.An organic basis exists for this type of disorder. b.Nurses perceive clients with this disorder as easy to care for. c.No known link exists between this disorder and early childhood loss or trauma. d.This disorder results in a split in the personality causing a lack of integration.

a.An organic basis exists for this type of disorder. The theory of structural dissociation of the personality proposes that patients with complex trauma have different parts of their personality, the apparently normal part and the emotional part, that are not fully integrated with each other. Each part has its own responses, feelings, thoughts, perceptions, physical sensations, and behaviors. These different parts may not be aware of each other, with only one dominant personality operating depending on the situation and circumstance of the moment.

Which of the following would be assessed as a negative symptom of schizophrenia? a.Anhedonia b.Hostility c.Agitation d.Hallucinations

a.Anhedonia Negative symptoms refer to deficits that characterize schizophrenia. They include the crippling symptoms of affective blunting (lack of facial expression), anergia (lack of energy), anhedonia (inability to experience happiness), avolition (lack of motivation), poverty of content of speech, poverty of speech, and thought blocking

Which of the following classifications of medication may be prescribed in intermittent explosive disorder? a.Anticonvulsants b.Psychostimulants c.Antianxiety agents such as benzodiazepines d.Monoamine oxidase (MAO) inhibitors

a.Anticonvulsants Although considered off-label use, anticonvulsants may reduce outbursts and contribute to mood stabilization. The other options are incorrect for use in intermittent explosive disorder

As an adult, a client who has been diagnosed with childhood-onset conduct disorder is at high risk for developing which comorbid disorder? a.Antipersonality disorder b.Obsessive-compulsive disorder c.Kleptomania d.Depression

a.Antipersonality disorder Individuals with childhood-onset conduct disorder are more likely to have problems that persist through adolescence, and without intensive treatment, they develop antisocial personality disorder as adults.

The nurse is preparing to set goals for a 10-year-old diagnosed with an impulse control disorder. To best ensure the expected therapeutic outcomes, the nurse includes goals that focus on what client need? a.Client centered and includes the client's input b.Age appropriate and achievable in a short period of time c.Simple and easily defined d.Family centered and long term in nature

a.Client centered and includes the client's input Whenever possible, outcomes should be client centered and agreed upon by both the nurse and the client or the client's designee. While the other options may be appropriate, they are not the priority.

What is a possible outcome criterion for a client diagnosed with anxiety disorder? a.Client demonstrates effective coping strategies. b.Client reports reduced hallucinations. c.Client reports feelings of tension and fatigue. d.Client demonstrates persistent avoidance behaviors.

a.Client demonstrates effective coping strategies.

Which coping mechanism is used excessively by clients diagnosed with bulimia nervosa to cope with their obsession with their body image? a.Denial b.Humor c.Altruism d.Projection

a.Denial Denial of incongruence between body reality, body ideal, and body presentation is the mainstay of the client diagnosed with bulimia nervosa.

What information should the nurse give to the family of a client who has had a dissociative episode? a.Dissociation is a method for coping with severe stress. b.Dissociation suggests the possibility of early dementia. c.Brief periods of psychotic behavior may occur. d.Ways to intervene to prevent self-mutilation and suicide attempts.

a.Dissociation is a method for coping with severe stress. Childhood physical, sexual, or emotional abuse and other traumatic events are associated with adults experiencing dissociative symptoms.

A client experiencing a panic attack keeps repeating, "Im dying, I can't breathe.". What action by the nurse should be most therapeutic initially? a.Encouraging the client to take slow, deep breaths b.Verbalizing mild disapproval of the anxious behavior c.Asking the client what he means when he says "I am dying." d.Offering an explanation about why the symptoms are occurring

a.Encouraging the client to take slow, deep breaths Slow diaphragmatic breathing can induce relaxation and reduce symptoms of anxiety. Often the nurse has to tell the client to "breathe with me" and keep the client focused on the task. The slower breathing also reduces the threat of hypercapnia with its attendant symptoms. The client needs help to regain composure and stabilize vital signs; the only option that addresses these issues is the correct option.

Which of the following describe the symptoms of the manic phase of bipolar disorder? Select all that apply. a.Excessive energy b.Fatigue and increased sleep c.Low self-esteem d.Pressured speech e.Purposeless movement f.Racing thoughts g.Withdrawal from environment h.Distractibility

a.Excessive energy d.Pressured speech e.Purposeless movement f.Racing thoughts h.Distractibility

A client diagnosed with conduct disorder craves what experience? a.Excitement without concern for possible negative outcomes b.Control of situations and constantly strategizes for such power c.Friendship but from those older than themselves d.Material possessions but lacks focus and direction

a.Excitement without concern for possible negative outcomes People with conduct disorder crave excitement and do not worry as much about consequences as other people do. None of the other options demonstrates a need associated with conduct disorder.

The client who will most likely respond well to drug therapy for the management of compulsive deviant sexual behavior is one with which diagnosis? a.Exhibitionism b.Antisocial personality disorder c.Low sexual drive d.Fetishism

a.Exhibitionism Libido and compulsive deviant sexual behavior is best managed pharmacologically in individuals with high sexual drive such as exhibitionists.

A physician describes a client as "malingering." The nurse knows this means that the client is demonstrating which behavior? a.Falsely claiming to have symptoms. b.Experiencing symptoms that cannot be explained medically. c.Experiencing symptoms that have a physiological basis. d.Seeking medication to ease pain of psychological origin.

a.Falsely claiming to have symptoms. Malingering is a consciously motivated act to deceive based on the desire for material gain. The symptoms described are nonexisting and so none of the other options are correct statements of behavior

Which side effects of lithium can be expected at therapeutic levels? a.Fine hand tremor and polyuria b.Nausea and thirst c.Coarse hand tremor and gastrointestinal upset d.Ataxia and hypotension

a.Fine hand tremor and polyuria The fact that fine hand tremor and polyuria are present at therapeutic levels is quite annoying to some clients. These and other side effects are factors in noncompliance.

A depressed client is likely to report a sleep disorder that includes which characteristics? a.Frequent awakenings during the night b.Nightmares c.Difficulty falling asleep d.Sleepwalking

a.Frequent awakenings during the night Depressed clients often report normal sleep onset, followed by repeated awakenings during the second half of the night. While the other options may occur, they are not the most frequent characteristic.

A client hospitalized with anorexia nervosa has a weight that is 65% of normal. For this client, what is a realistic short-term goal for the first week of hospitalization regarding the physical impact of his/her weight? a.Gain a maximum of 3 lb. b.Develop a pattern of normal eating behavior. c.Discuss fears and feelings about gaining weight. d.Verbalize awareness of the sensation of hunger.

a.Gain a maximum of 3 lb. The critical outcome during hospitalization for anorexia nervosa is weight gain. A maximum of 3 pounds weekly is considered sufficient initially. Too-rapid weight gain can cause pulmonary edema. While all the remaining goals are appropriate, none have the physical focus that is the initial priority.

When providing possible interventions to promote the safety of a client reporting symptoms of somnambulism, the nurse should include which intervention? a.Gating the stairways b.Sleeping on a mattress placed on the floor c.Regular bedtime dose of a benzodiazepine d.Avoiding the use of serotonergic medications

a.Gating the stairways Somnambulism or sleep walking can result in falls; gating the stairways may minimize that risk.

A woman complains to the nurse, "It hurts when I have intercourse." Which diagnostic term applies? a.Genitopelvic pain/penetration disorder b.Female sexual interest/arousal disorder c.Female orgasmic disorder d.Voyeurism

a.Genitopelvic pain/penetration disorder

Which behavior would be characteristic of a client during a manic episode? a.Going rapidly from one activity to another b.Taking frequent rest periods and naps during the day c.Being unwilling to leave home to see other people d.Watching others intently and talking little

a.Going rapidly from one activity to another Hyperactivity and distractibility are basic to manic episodes. None of the other options demonstrate such characteristics.

The symptoms of an adjustment disorder can include characteristics? Select all that apply. a.Guilt b.Social withdrawal c.Overachieving d.Anger e.Depression

a.Guilt b.Social withdrawal d.Anger e.Depression In contrast to acute stress disorder responses, which are quite severe and include anxiety and fear, symptoms of an adjustment disorder can run the gamut of all forms of distress including guilt, depression, and anger. These feelings may be combined with other manifestations of distress, including physical complaints, social withdrawal, or work or academic inhibition

Which of the following statements are true regarding childhood-onset conduct disorder? Select all that apply. a.It is more commonly diagnosed in males. b.It is characterized by feelings of remorse and regret. c.It is usually diagnosed in late teen years. d.It is characterized by disregard for the rights of others. e.Those with conduct disorder rationalize their aggressive behaviors. f.It is usually outgrown by early adulthood.

a.It is more commonly diagnosed in males. d.It is characterized by disregard for the rights of others. e.Those with conduct disorder rationalize their aggressive behaviors. Childhood-onset conduct disorder is more common in male patients and is seen before the age of 10 years. Hallmarks include disregard for the rights of others, physical aggression, poor peer relationships, and lack of feelings of guilt or remorse. The other options are the opposite of what is correct.

What is the first-line drug used to treat mania? a.Lithium carbonate b.Carbamazepine c.Lamotrigine d.Clonazepam

a.Lithium carbonate Lithium, a mood stabilizer, is the first-line drug for use in treating bipolar disorder. The other options are prescribed to manage other related symptoms of bipolar disorder

A 5-year-old who consistently omits the sound for 'r' and 's' when speaking is demonstrating which type of disorder? a.Speech b.Language c.Social communication d.Specific learning

a.Speech Speech disorders are marked by problems in making sounds. Children may have trouble making certain sounds, or they may distort, add, or omit sounds. Such patterns are not associated with any of the other options.

An 8-year-old patient is newly diagnosed with attention deficit hyperactivity disorder (ADHD). It is important that the parents be educated to the fact that symptoms will take which form? (Select all that apply). of, inattention, and impulsivity have to be apparent: Select all that apply. a.Low frustration tolerance b.Poor school performance c.Impulsive behaviors d.Easily intimidated e.Mood swings

a.Low frustration tolerance b.Poor school performance c.Impulsive behaviors e.Mood swings Individuals with ADHD show an inappropriate degree of inattention, impulsiveness, and hyperactivity. Attention problems and hyperactivity contribute to low frustration tolerance, temper outbursts, labile moods, poor school performance, peer rejection, and low self-esteem. ADHD is not generally characterized by meekness or by being easily intimidated

The client experiencing bulimia differs from the client diagnosed with anorexia nervosa by exhibiting which characteristic? a.Maintaining a normal weight b.Holding a distorted body image c.Doing more rigorous exercising d.Purging to keep weight down

a.Maintaining a normal weight Many bulimics are at or near normal weight, whereas clients with anorexia nervosa are underweight

Which statement about antidepressant medications, in general, can serve as a basis for client and family teaching? a.Onset of action is from 1 to 3 weeks or longer. b.They tend to be more effective for men. c.Recent memory impairment is commonly observed. d.They often cause the client to have diurnal variation.

a.Onset of action is from 1 to 3 weeks or longer. A drawback of antidepressant drugs is that improvement in mood may take 1 to 3 weeks or longer. None of the other options provide correct information regarding antidepressant medications.

Inability to leave one's home because of avoidance of severe anxiety suggests the existence of which anxiety disorder? a.Panic attacks with agoraphobia b.Obsessive-compulsive disorder c.Posttraumatic stress response d.Generalized anxiety disorder

a.Panic attacks with agoraphobia Panic disorder with agoraphobia is characterized by recurrent panic attacks combined with agoraphobia. Agoraphobia involves intense, excessive anxiety about being in places or situations from which escape might be difficult or embarrassing or in which help might not be available if a panic attack occurred.

Which statement is true of the eating disorder referred to as bulimia? a.Patients with bulimia often appear at a normal weight. b.Patients with bulimia binge eat but do not engage in compensatory measures. c.Patients with bulimia severely restrict their food intake. d.One sign of bulimia is lanugo.

a.Patients with bulimia often appear at a normal weight. Patients with bulimia are often at or close to ideal body weight and do not appear physically ill. The other options do not refer to bulimia but rather refer to signs of binge eating disorder and anorexia nervosa.

When working with a client demonstrating impulse control disorders, which nursing interventions have initial priority? Select all that apply. a.Providing a safe environment b.Establishing a therapeutic nurse-client relationship c.Setting and enforcing limits d.Confronting the client concerning the disruptive behavior e.Presenting appropriate expectations

a.Providing a safe environment b.Establishing a therapeutic nurse-client relationship c.Setting and enforcing limits e.Presenting appropriate expectations The most important interventions with this population are to promote a climate of safety for the patient and for others, establish rapport with the patient, and set limits and expectations. Confronting behaviors is not an initial priority.

The most common course of schizophrenia is an initial episode followed by what course of events? a.Recurrent acute exacerbations and deterioration b.Recurrent acute exacerbations c.Continuous deterioration d.Complete recovery

a.Recurrent acute exacerbations and deterioration Schizophrenia is usually a disorder marked by an initial episode followed by recurrent acute exacerbations. With each relapse of psychosis, an increase in residual dysfunction and deterioration occurs.

Providing care to a client diagnosed with a somatization disorder can be frustrating owing to the client's lack of an organic illness. In order to best manage this barrier to care the staff should implement which personal intervention? a.Regularly discuss their feelings about the client during the unit's interprofessional care meetings. b.Attend in-services that focus on the various aspects of somatic disorders. c.Rotate care of the client among the entire nursing department staff to minimize the frustration. d.Provide a unified approach to the client's behavior so as to manage and lessen the barrier itself.

a.Regularly discuss their feelings about the client during the unit's interprofessional care meetings. It is helpful for health care workers, no matter the setting, to discuss responses to these patients in conferences with other health care members to allow for expression of feelings and, ultimately, to provide for consistent care. While the other options are appropriate, none are as staff oriented as the correct option.

Which factor can reduce the vulnerability of a child to etiological influences predisposing to the development of psychopathology? a.Resilience b.Malnutrition c.Child abuse d.Having a depressed parent

a.Resilience Resilience refers to developing and using certain characteristics that help a child to handle the stresses of a difficult childhood without developing mental problems. Resilient children can adapt to changes in the environment, form nurturing relationships with adults other than their parents, distance themselves from the emotional chaos of the family, and have social intelligence and the ability to use problem-solving skills.

What is the priority nursing diagnosis for a hyperactive manic client during the acute phase of treatment? a.Risk for injury b.Ineffective role performance c.Risk for other-directed violence d.Impaired verbal communication

a.Risk for injury Risk for injury is high, related to the client's hyperactivity and poor judgment. Safety is always the priority when considering client care.

Which statement accurately applies to exhibitionism? a.Seldom a precursor to sexual assault or rape. b.Generally viewed as a victimless crime. c.Rarely prosecuted. d.Generally viewed as an illness by the courts.

a.Seldom a precursor to sexual assault or rape. Exhibitionism is generally done more for shock value, and actual physical contact is rarely sought.

A client confides to the nurse that she is sexually excited by dominating her partner and achieves orgasm only when she humiliates her partner. This admission supports which sexual disorder? a.Sexual sadism b.Orgasmic disorder c.Sexual pain disorder d.Immature sexual gratification

a.Sexual sadism Sexual sadism involves the need to give psychological or physical pain to achieve sexual gratification.

A cultural characteristic that may be observed in a teenage, female Hispanic client in times of stress would include what behavior? a.Suddenly tremble severely b.Exhibit stoic behavior c.Report both nausea and vomiting d.Laugh inappropriately

a.Suddenly tremble severely Ataque de nervios (attack of the nerves) is a culture-bound syndrome that is seen in undereducated, disadvantaged females of Hispanic ethnicity

A client reports to the nurse that falling asleep can often take hours. Which intervention should the nurse implement? a.Teach the client how to do progressive relaxation. b.Advise the client to drink an ounce or two of brandy at bedtime. c.Suggest that the client seek a referral for polysomnography. d.Point out that reducing stress at work would be advisable.

a.Teach the client how to do progressive relaxation. Progressive relaxation relaxes muscle groups sequentially and generates a state of pleasant comfort and ease, a natural prelude to sleep. It is inappropriate to encourage the use of alcohol as a sleep aid since it is known to decrease stage 3 sleep. A referral for a sleep test is premature. While stress reduction is appropriate, concentrating only on work-related stress is not effective

The nurse working with clients diagnosed with eating disorders can help families develop effective coping mechanisms by implementing which intervention? a.Teaching the family about the disorder and the client's behaviors b.Stressing the need to suppress overt conflict within the family c.Urging the family to demonstrate greater caring for the client d.Encouraging the family to use their usual social behaviors at meals

a.Teaching the family about the disorder and the client's behaviors Families need information about specific eating disorders and the behaviors often seen in clients with these disorders. This information can serve as a basis for additional learning about how to support the family member. While the other options may be appropriate for specific client families, they are not as fundamental as the correct option.

When a nurse assesses the style of behavior a child habitually uses to cope with the demands and expectations of the environment, he or she is assessing characteristic? a.Temperament b.Resilience c.Vulnerability d.Cultural assimilation

a.Temperament Temperament is the behavior the child habitually uses to cope with the environment. It is a constitutional factor thought to be genetically determined. It may be modified by the parent-child relationship.

When attempting to determine the cause of low sexual drive in either a male or female client, the nurse can expect evaluation of the client's serum level of which hormone? a.Testosterone b.Estrogen c.Thyroxin d.Insulin

a.Testosterone Testosterone, present in both males and females, appears to be essential to sexual desire in both men and women.

Generally, which statement regarding ego defense mechanisms is true? a.They often involve some degree of self-deception. b.They are rarely used by mentally healthy people. c.They seldom make the person more comfortable. d.They are usually effective in resolving conflicts.

a.They often involve some degree of self-deception. Most ego defense mechanisms, with the exception of the mature defenses, alter the individual's perception of reality to produce varying degrees of self-deception.

A 9-year-old patient has been diagnosed with an intellectual development disorder (IDD). Which assessment findings support this diagnosis? Select all that apply. a.Unable to explain the phrase, "Raining cats and dogs" b.Reads below age level c.Is capable of providing effective oral self care d.Enjoy interacting with developmentally similar peers e.Physically lashes out when frustrated

a.Unable to explain the phrase, "Raining cats and dogs" b.Reads below age level e.Physically lashes out when frustrated IDD is characterized by severe deficits in three major areas of functioning: intellectual, social, and managing daily life. These children demonstrate difficulty with self care and with almost any social interactions.

When the nurse remarks to a depressed client, "I see you are trying not to cry. Tell me what is happening." The nurse should be prepared to implement which intervention? a.Waiting quietly for the client to reply b.Prompting the client if the reply is slow c.Repeating the question if the client does not answer promptly d.Reviewing the client's medical record to support the client's response

a.Waiting quietly for the client to reply Depressed clients think slowly and take long periods to formulate answers and respond. The nurse must be prepared to wait for a reply.

After stabilization of symptoms, what is the primary focus of treatment for a client diagnosed with anorexia nervosa? a.Weight restoration b.Improving interpersonal skills c.Learning effective coping methods d.Changing family interaction patterns

a.Weight restoration Weight restoration is the priority goal of treatment for the client with anorexia nervosa because health is seriously threatened by the underweight status. The other options are addressed are secondary to the physiological goal of weight restoration.

When a client reports that lithium causes an upset stomach, the nurse should make which suggestion associated with taking the medication? a.With meals b.With an antacid c.30 minutes before meals d.2 hours after meals

a.With meals Many clients find that taking lithium with or shortly after meals minimizes gastric distress. None of the other options present accurate information.

What action should the nurse take on learning that a manic client's serum lithium level is 1.8 mEq/L? a.Withhold medication and notify the physician. b.Continue to administer medication as ordered. c.Advise the client to limit fluids for 12 hours. d.Advise the client to curtail salt intake for 24 hours.

a.Withhold medication and notify the physician. The client's lithium level has exceeded desirable limits. Additional doses of the medication should be withheld and the physician notified. None of the other options are accurate interventions.

An acute phase nursing intervention aimed at reducing hyperactivity is demonstrated by which intervention? a.Writing in a diary b.Exercising in the gym c.Directing unit activities d.Orienting a new client to the unit

a.Writing in a diary Manic clients often respond well to the invitation to write. They will fill reams of paper. While writing they are less physically active. None of the remaining options presents this opportunity to reduce physical activity

A client diagnosed with paranoid schizophrenia tells the nurse, "I have to get away. The volmers are coming to execute me." The term "volmers" can be assessed as a.a neologism. b.clang association. c.blocking. d.a delusion.

a.a neologism. A neologism is a newly coined word that has meaning only for the client. None of the other options fit this description

Which assessment finding represents a negative symptom of schizophrenia? a.apathy b.delusion c.motor tic d.hallucination

a.apathy

Which anticonvulsant medication might be prescribed for a patient with bipolar disorder? a.divalproex sodium (Depakote) b.clonazepam (Klonopin) c.olanzapine (Zyprexa) d.lithium (Lithoid)

a.divalproex sodium (Depakote)

A patient with schizophrenia says, "There are worms under my skin eating the hair follicles." How would you classify this assessment finding? a.positive symptom b.negative symptom c.cognitive symptom d.depressive symptom

a.positive symptom

A patient with major depression walks and moves slowly. Which term should the nurse use to document this finding? a.psychomotor retardation b.psychomotor agitation c.vegetative sign d.anhedonia

a.psychomotor retardation

Which statement made by a 9-year-old child after hitting a classmate is a typical comment associated with childhood conduct disorder? a. "I'm sorry, I won't hit him again." b. "He deserved it for being a sissy." c. "I didn't think I hit him very hard." d. "He hit me first. You just didn't see it."

b. "He deserved it for being a sissy."

Cognitive-behavioral therapy is going well when a 12-year- old patient in therapy reports to the nurse practitioner: a. "I was so mad I wanted to hit my mother." b. "I thought that everyone at school hated me. That's not true. Most people like me and I have a friend named Todd." c. "I forgot that you told me to breathe when I become angry." d. "I scream as loud as I can when the train goes by the house."

b. "I thought that everyone at school hated me. That's not true. Most people like me and I have a friend named Todd."

Which patient statement acknowledges the characteristic behavior associated with a diagnosis of pica? a. "Nothing could make me drink milk." b. "I'm ashamed of it, but I eat my hair." c. "I haven't eaten a green vegetable since I was 3 years old." d. "I regurgitate and re-chew my food after almost every meal."

b. "I'm ashamed of it, but I eat my hair."

Which statement(s) made by the nurse demonstrates an understanding of the effective use of relaxation therapy for anxiety management? Select all that apply. a. "Relaxation therapy's main goal is to prevent exhaustion by removing muscle tension." b. "Muscle relaxation promotes the relaxation response." c. "Show me how you learned to deep breathe in yesterday's therapy session." d. "You've said that going to group makes you nervous so let's start relaxing now." e. "I've given you written descriptions of the various relaxation exercises for you to review."

b. "Muscle relaxation promotes the relaxation response." c. "Show me how you learned to deep breathe in yesterday's therapy session." d. "You've said that going to group makes you nervous so let's start relaxing now." e. "I've given you written descriptions of the various relaxation exercises for you to review."

Which assessment question asked by the nurse demonstrates an understanding of comorbid mental health conditions associated with major depressive disorder? Select all that apply. a. "Do rules apply to you?" b. "What do you do to manage anxiety?" c. "Do you have a history of disordered eating?" d. "Do you think that you drink too much?" e. "Have you ever been arrested for committing a crime?"

b. "What do you do to manage anxiety?" c. "Do you have a history of disordered eating?" d. "Do you think that you drink too much?"

Which event experienced in the patient's childhood increases the risk of the development of behaviors associated with intermittent explosive disorder? a. Orphaned at age 4 b. Physically abused from ages 3 to 10 c. Born with a chronic congenital disorder d. One parent was diagnosed with obsessive-compulsive disorder

b. Physically abused from ages 3 to 10

Which characteristic identified during an assessment serves to support a diagnosis of disruptive mood dysregulation disorder? Select all that apply. a. Female b. 7 years old c. Comorbid autism diagnosis d. Outbursts occur at least once a week e. Temper tantrums occur at home and in school

b. 7 years old c. Comorbid autism diagnosis e. Temper tantrums occur at home and in school

Substance abuse is often present in people diagnosed with bipolar disorder. Laura, a 28-year-old with a diagnosis of bipolar disorder, drinks alcohol instead of taking her prescribed medications. The nurse caring for this patient recognizes that: a. Anxiety may be present. b. Alcohol ingestion is a form of self-medication. c. The patient is lacking a sufficient number of neurotransmitters. d. The patient is using alcohol because she is depressed.

b. Alcohol ingestion is a form of self-medication.

Tatiana has been hospitalized for an acute manic episode. On admission the nurse suspects lithium toxicity. What assessment findings would indicate the nurse's suspicion as correct? a. Shortness of breath, gastrointestinal distress, chronic cough b. Ataxia, severe hypotension, large volume of dilute urine c. Gastrointestinal distress, thirst, nystagmus d. Electroencephalographic changes, chest pain, dizziness

b. Ataxia, severe hypotension, large volume of dilute urine

The care plan of a patient diagnosed with a somatic disorder includes the nursing diagnosis ineffective coping. Which patient behavior demonstrates a successful outcome for that nursing diagnosis? a. Showers and dresses in clean clothes daily b. Calls a friend to talk when feeling lonely c. Spends more time talking about pain in her abdomen d. Maintains focus and concentration

b. Calls a friend to talk when feeling lonely

Adolescents often display fluctuations in mood along with undeveloped emotional regulation and poor tolerance for frustration. Emotional and behavioral control usually increases over the course of adolescence due to: a. Limited executive function b. Cerebellum maturation c. Cerebral stasis and hormonal changes d. A slight reduction in brain volume

b. Cerebellum maturation

Obesity can be the end result of a binge-eating disorder. The nurse understands that the best treatment option in persons with a binge-eating disorder promotes: a. Bariatric surgery b. Coping strategies c. Avoidance of public eating d. Appetite suppression medications

b. Coping strategies

Some cultures have lower rates of diagnosed conduct disorders than observed in Western societies. The lower rate of incidence may be contributed to: a. Strict parenting with corporal punishment b. Cultural expression of anger as normal behavior c. Parents' limited tolerance for externalizing behavior d. Widespread acceptance of conduct disorders

b. Cultural expression of anger as normal behavior

The biological approach to treating depression with electrodes surgically implanted into specific areas of the brain to stimulate the regions identified to be underactive in depression is: a. Transcranial magnetic stimulation b. Deep brain stimulation c. Vagus nerve stimulation d. Electroconvulsive therapy

b. Deep brain stimulation

Which medication should the nurse be prepared to educate patients on when they are prescribed a selective serotonin reuptake inhibitor (SSRI) for panic attacks? a. Alprazolam (Xanax) b. Fluoxetine (Prozac) c. Clonazepam (Klonopin) d. Venlafaxine (Effexor)

b. Fluoxetine (Prozac)

A male patient calls to tell the nurse that his monthly lithium level is 1.7 mEq/L. Which nursing intervention will the nurse implement initially? a. Reinforce that the level is considered therapeutic. b. Instruct the patient to hold the next dose of medication and contact the prescriber. c. Have the patient go to the hospital emergency room immediately. d. Alert the patient to the possibility of seizures and appropriate precautions.

b. Instruct the patient to hold the next dose of medication and contact the prescriber.

Samantha is a new patient to the mental health clinic and is seeking assistance for what she describes as "severe anxiety." In addition to daily self-medicating with alcohol, Samantha describes long-term use of herbal kava. The nurse knows that kava is associated with inhibiting P450 and orders which of the following tests? a. Electrocardiogram b. Liver enzymes c. Glomerular filtration rate d. Complete blood count

b. Liver enzymes

Pedophilic disorder is the most common paraphilic disorder where adults who have a primary or exclusive sexual preference for prepubescent children. A subset of this disorder is termed hebephilia and is defined as attraction to: a. Infants b. Pubescent individuals c. Teens between the ages of 15 and 19 d. Males only

b. Pubescent individuals

April, a 10-year-old admitted to inpatient pediatric care, has been getting more and more wound up and is losing self-control in the day room. Time-out does not appear to be an effective tool for April to engage in self-reflection. April's mother admits to putting her in time-out up to 20 times a day. The nurse recognizes that: a. Time-out is an important part of April's baseline discipline. b. Time-out is no longer an effective therapeutic measure. c. April enjoys time-out, and acts out to get some alone time. d. Time-out will need to be replaced with seclusion and restraint.

b. Time-out is no longer an effective therapeutic measure.

The care plan of a male patient diagnosed with a dissociative disorder includes the nursing diagnosis ineffective coping. Which behavior demonstrated by the patient supports this nursing diagnosis? a. Has no memory of the physical abuse he endured. b. Using both alcohol and marijuana. c. Often reports being unaware of surroundings. d. Reports feelings of "not really being here."

b. Using both alcohol and marijuana.

A new psychiatric technician mentions to the nurse, "Depression seems to be a disease of old people. All the depressed clients on the unit are older than 60 years." How should the nurse respond to this statement? a."That is a good observation. Depression does mostly strike people older than 50 years." b."Depression is seen in people of all ages, from childhood to old age." c."Depression is most often seen among the middle adult age group." d."The age of onset for most depressive episodes is given as 18 years."

b."Depression is seen in people of all ages, from childhood to old age." Depression can occur at any age. Children, adolescents, adults, and the elderly may all experience depression

A client reports symptomatology that supports the diagnosis of sleep paralysis. The nurse effectively assesses the client by asking which question? a."Do you ever have nightmares?" b."Have you ever fallen asleep while driving?" c."Do you have a history of obsessive compulsive behavior?" d."Is it difficult for you to fall asleep?"

b."Have you ever fallen asleep while driving?" Clients with sleep paralysis can often also exhibit symptoms of narcolepsy such as extreme sleepiness resulting in falling asleep at inappropriate times. None of the other questions are directed toward this disorder.REF

Which statement would best show acceptance of a depressed, mute client? a."I will be spending time with you each day to try to improve your mood." b."I would like to sit with you for 15 minutes now and again this afternoon." c."Each day we will spend time together to talk about things that are bothering you." d."It is important for you to share your thoughts with someone who can help you evaluate your thinking."

b."I would like to sit with you for 15 minutes now and again this afternoon." Spending time with the client without making demands is a good way to show acceptance. While not inappropriate, the other options are less accepting.

Which statement, made by a client diagnosed with dissociative identity disorder, demonstrates effective understanding in response to the question, "What exactly are the 'alters'? your provider told you about?" illustrates that the education you provided has been effective? a."So, alters are based in mysticism and religiosity, such as demons." b."So, alters are separate personalities with their own characteristics that take over during stress." c."So, alters are never aware of each other." d."So, alters are just like me, but they have no memory of the trauma I went through."

b."So, alters are separate personalities with their own characteristics that take over during stress." Dissociative identity disorder appears to be associated with at least two dissociative identity states: one is a state or personality that functions on a daily basis and blocks access and responses to traumatic memories, and another state (also referred to as an alter state) is fixated on traumatic memories. Each alter has its own memories, behavior patterns, and characteristics. Transition from one personality to another (switching) occurs during times of stress. The other responses are incorrect, because alters may be aware of the existence of each other to some degree, and alters are not just like the host—they have different behaviors and memories.

A client has been diagnosed with gender identity disorder. The nurse can expect that the client will evidence which characteristic? a.Intense sexual urges focused on an object b.Discomfort with biological gender c.Self-humiliation during the sexual act d.Inability to maintain sexual arousal

b.Discomfort with biological gender Gender identity disorder involves the lack of a match between biological gender and psychological gender anxiety. The client will state that he is a woman who was mistakenly given a man's body.

The mother of a 3-year-old boy just diagnosed with autism spectrum is tearful and states, "The doctor said we need to start therapy right away. I just don't understand how helpful it will be—he's only 3 years old!" What response should the nurse provide to the mother's statement? a."You are right, 3 years old is very young to start therapy, but it will make you feel better to be doing something." b."Starting him on treatment now gives Taylor a much greater chance for a productive life." c."If your child starts therapy now, he will be able to stop therapy sooner." d."If you have questions, its best to ask the doctor."

b."Starting him on treatment now gives Taylor a much greater chance for a productive life." Early intervention for children with autism can greatly enhance their potential for a full, productive life. 3 years old is not too young to start therapy since the sooner therapy is started, the better the outcome. The patient will most likely not be able to stop therapy as interventions will continue indefinitely. Telling the mother to ask her provider abdicates the nurse's responsibility to provide education to patients and families.

An adolescent faints during gym class. She is grossly underweight, wears baggy clothes, and her skin is dry. To further assess for anorexia nervosa, the school nurse should ask a."Do you often wear heavy clothing in warm weather?" b."When was your last menstrual period?" c."Do you use any drugs or alcohol?" d."Do you ever lose lapses of time?"

b."When was your last menstrual period?"

Which question would be a priority when assessing for symptoms of major depression? a."Tell me about any special powers you believe you have." b."You look really sad. Have you ever thought about harming yourself?" c."Your family says you never stop. How much sleep do you get?" d."Do you ever find you don't remember where you've been or what you've done?"

b."You look really sad. Have you ever thought about harming yourself?"

Ever since participating in a village raid where explosives were used, a military veteran has been unable to walk. After all diagnostic testing were negative for any physical abnormalities, the client was diagnosed with conversion disorder. What is the nurse's best response when asked by the client, "Why can't I walk?" a."Your legs don't work because your brain is screwed up." b."Your emotional distress is being expressed as a physical symptom." c."You are making up your symptoms as a cry for help." d."You are overly anxious about having a severe illness."

b."Your emotional distress is being expressed as a physical symptom." Conversion disorder is attributed to channeling of emotional conflicts or stressors into physical symptoms. Telling the patient her brain is "screwed up" is unprofessional and does not give any useful education. Symptoms of conversion disorder are not within the patient's voluntary control. Being overly anxious about having a severe illness describes illness anxiety disorder.

Which client is most likely to initially demonstrate behaviors suggesting a somatic disorder? a.13-year-old male b.23-year-old female c.33-year-old male d.43-year-old female

b.23-year-old female

What symptom can the nurse expect a client diagnosed with depersonalization disorder to manifest? a.Aimless wandering with confusion and disorientation b.A feeling of detachment from one's body or mental processes c.Existence of two or more personalities that take control of behavior d.Worry about having a serious disease based on symptom misinterpretation

b.A feeling of detachment from one's body or mental processes Depersonalization is characterized by a sense of unreality or self-estrangement. None of the other options present an expected characteristic of depersonalization disorder.

An obsession is defined as what? a.Thinking of an action and immediately taking the action b.A recurrent, persistent thought or impulse c.An intense irrational fear of an object or situation d.A recurrent behavior performed in the same manner

b.A recurrent, persistent thought or impulse Obsessions are thoughts, impulses, or images that persist and recur so that they cannot be dismissed from the mind.

What is a desired outcome for the maintenance phase of treatment for a manic client? a.Exhibit optimistic, energetic, playful behavior. b.Adhere to follow-up medical appointments. c.Take medication more than 50% of the time. d.Use alcohol to moderate occasional mood "highs."

b.Adhere to follow-up medical appointments. The client would be living in the community during the maintenance phase. Keeping follow-up appointments is highly desirable. None of the other options are accurate.

A client's daughter states, "My mother lives with me since my dad died 6 months ago. For the past couple of months, every time I need to leave the house for work or anything else, Mom becomes extremely anxious and cries that something terrible is going to happen to me. She seems OK except for these times, but it's affecting my ability to go to work." This information supports that the client may be experiencing which anxiety-related disorder? a.Panic disorder b.Adult separation anxiety disorder c.Agoraphobia d.Social anxiety disorder

b.Adult separation anxiety disorder People with separation anxiety disorder exhibit developmentally inappropriate levels of concern over being away from a significant other. There may also be fear that something horrible will happen to the other person. Adult separation anxiety disorder may begin in childhood or adulthood. The scenario doesn't describe panic disorder. Agoraphobia is characterized by intense, excessive anxiety or fear about being in places or situations from which escape might be difficult or embarrassing or in which help might not be available. Social anxiety disorder, also called social phobia, is characterized by severe anxiety or fear provoked by exposure to a social or a performance situation that will be evaluated negatively by others.

What defense mechanisms can only be used in healthy ways? a.Suppression and humor b.Altruism and sublimation c.Idealization and splitting d.Reaction formation and denial

b.Altruism and sublimation Altruism and sublimation are known as mature defenses. They cannot be used in unhealthy ways. Altruism results in resolving emotional conflicts by meeting the needs of others, and sublimation substitutes socially acceptable activity for unacceptable impulses.

A client diagnosed with bulimia nervosa uses enemas and laxatives to purge to maintain weight. What is the likely physiological outcome of this practice? a.Increase in the red blood cell count b.Disruption of the fluid and electrolyte balance c.Elevated serum potassium level d.Elevated serum sodium level

b.Disruption of the fluid and electrolyte balance Disruption of the fluid and electrolyte balance is usually the result of excessive use of enemas and laxatives. There would be a decrease in potassium and sodium levels while the concentration of but not actual red cell count would be affected.

A 12-year-old male patient diagnosed with Tourette's disorder is visiting his provider. The nurse will prepare medication teaching on which class of medication to help manage the tics associated with this disorder Select all that apply. a.Mood stabilizers b.Antianxiety agents c.Anticholinesterase inhibitors d.First-generation antipsychotics e.Second-generation antipsychotics

b.Antianxiety agents d.First-generation antipsychotics e.Second-generation antipsychotics Drugs with Food and Drug Administration (FDA) approval for treating tics are the first-generation antipsychotics haloperidol and pimozide, and the second-generation antipsychotic aripiprazole.Clonidine hydrochloride, an alpha 2-adrenergic agonist, used to treat hypertension, is also prescribed for tics. While less effective and far slower acting than the antipsychotics, it has fewer side effects. The antianxiety drug clonazepam (Klonopin) is used as a supplement to other medications. It may work by reducing anxiety and resultant tics. The other options are not used or approved for the treatment of Tourette's disorder

A 16-year-old patient being treated for anorexia, has been prescribed medication to reduce compulsive behaviors regarding food now that ideal weight has been reached. Which class of medication is prescribed for this specific issue associated with eating disorders? a.Mood stabilizers b.Antidepressants c.Anxiolytics d.Atypical antipsychotics

b.Antidepressants The antidepressant fluoxetine (Prozac, an SSRI) has proven useful in reducing obsessive-compulsive behavior after the patient has reached a maintenance weight. Anxiolytics would be prescribed for anxiety. Atypical antipsychotic agents may be helpful in improving mood and decreasing obsessional behaviors and resistance to weight gain. Mood stabilizers are not specifically used in treatment of eating disorders.

A 7-year-old, who is described as impulsive and hyperactive, tells the nurse, "I am a dummy, because I don't pay attention, and I can't read like the other kids." The nurse notes that these behaviors are most consistent with which diagnosis? a.Attention deficit disorder b.Attention deficit hyperactivity disorder c.Autism d.Conduct disorder

b.Attention deficit hyperactivity disorder The data are most consistent with attention deficit hyperactivity disorder (ADHD) as described in the DSM-5. The other options present with characteristics and behaviors that differ from those in the scenario.

Which comorbid conditions are commonly associated with oppositional defiant disorder? Select all that apply. a.Conversion disorder b.Attention deficit hyperactivity disorder (ADHD) c.Depression d.Anxiety e.Substance abuse

b.Attention deficit hyperactivity disorder (ADHD) c.Depression d.Anxiety

A client is diagnosed with generalized anxiety disorder (GAD). The nursing assessment supports this diagnosis when the client reports which information? a.Symptoms started right after being robbed at gunpoint. b.Being unable to work for the last 12 months. c.Eating in public makes the client extremely uncomfortable. d.Repeated verbalizing prayers results in a relaxed feeling.

b.Being unable to work for the last 12 months. GAD is characterized by symptomatology that lasts 6 months or longer.

Which social behavior is often a result of a child having been exposed to some form of abuse? a.Speech disorders b.Bullying others c.Eating disorders d.Delayed motor skills

b.Bullying others Children who have experienced abuse are at risk for identifying with their aggressor and may act out, bully others, become abusers, or develop dysfunctional interpersonal relationships in adulthood. None of the remaining options are as directly associated with abuse as bullying

The nurse can expect a client demonstrating typical manic behavior to be attired in clothing that includes with characteristics? a.Dark colored and modest b.Colorful and outlandish c.Compulsively neat and clean d.Ill-fitted and ragged

b.Colorful and outlandish Manic clients often manage to dress and apply makeup in ways that create a colorful, even bizarre, appearance. None of the remaining options meet that criteria.

The mother of a 6-year-old child expresses concern over the child's frequent temper outbursts. He deals with any frustration by bullying and hitting and seldom shows any remorse for his actions. The nurse who gathers this data will note that the child's behaviors are most consistent with which diagnosis? a.Social phobia b.Conduct disorder c.Oppositional defiant disorder d.Attention deficit hyperactivity disorder (ADHD)

b.Conduct disorder The data are most consistent with the aggressive pattern of childhood-onset conduct disorder of the aggressive type

What therapeutic intervention should be prescribed for a client diagnosed with a somatoform disorder? a.Steering conversation away from the client's feelings b.Conveying an interest in the client rather than in the symptoms c.Encouraging the client to use benzodiazepines liberally d.Encouraging the client to rely on the nurse to meet the client's needs

b.Conveying an interest in the client rather than in the symptoms When the nurse focuses on the client rather than on the symptoms, the client's self-worth and coping skills are enhanced. The discussion related to client feelings is a major focus of therapy. Neither of the remaining options serves to help the client identify the causes of the illness and so would not serve as effective interventions

A man continues to speak of his wife as though she were still alive, 3 years after her death. This behavior suggests the use of which ego defense mechanism? a.Altruism b.Denial c.Undoing d.Suppression

b.Denial Denial involves escaping unpleasant reality by ignoring its existence

What statement about the comorbidity of depression is accurate? a.Depression most often exists in an individual as a single entity. b.Depression is commonly seen in individuals with medical disorders. c.Substance abuse and depression are seldom seen as comorbid disorders. d.Depression may coexist with other disorders but is rarely seen with schizophrenia.

b.Depression is commonly seen in individuals with medical disorders. Depression commonly accompanies medical disorders. The other options are false statements.

What can be said about the comorbidity of anxiety disorders? a.Anxiety disorders generally exist alone. b.Depression may occur prior to onset of anxiety. c.Anxiety disorders virtually never coexist with mood disorders. d.Substance abuse disorders rarely coexist with anxiety disorders.

b.Depression may occur prior to onset of anxiety. In many instances, major depression may occur prior to the onset of panic disorder or may occur at the same time. Clinicians and researchers have clearly shown that anxiety disorders frequently co-occur with other psychiatric problems. Major depression often co-occurs and produces a greater impairment with poorer response to treatment.

Which of the following statements about dissociative disorders is true? a.Dissociative symptoms are under the person's conscious control. b.Dissociative symptoms are not under the person's conscious control. c.Dissociative symptoms are usually a cry for attention. d.Dissociative symptoms are always negative.

b.Dissociative symptoms are not under the person's conscious control. Dissociation is involuntary and results in failure of the normal control over a person's mental processes and normal integration of conscious awareness. The other responses are untrue

Which nursing intervention is appropriate for the management of intermittent explosive disorder? a.Providing intensive family therapy b.Establishing a trusting relationship with the patient c.Setting up loose boundaries so the patient will feel relaxed d.Limiting decision-making opportunities to avoid frustration

b.Establishing a trusting relationship with the patient rapport with the patient is essential in working to set goals, boundaries, and consequences, and providing opportunities for goal achievement. Intensive family therapy would not be a basic level RN intervention. Boundaries and structure are essential. Opportunities for patients to make good decisions and reach goals should be given, not limited.

A 17-year-old patient is admitted to the psychiatric unit after threatening his mother during an argument and is diagnosed with conduct disorder. Which of the following would be an appropriate short-term outcome for this patient? a.Engages in appropriate coping skills to manage stressors b.Expresses feelings c.Maintains self-control during hospitalization d.Mother will improve communication skills to interact with Eli.

b.Expresses feelings Expressing feelings is an appropriate short-term outcome and would be a good start to working with the patient to establish rapport, develop coping skills, and set goals. Engaging in appropriate coping skills and maintaining self-control are desired outcomes. Outcomes for the patient are being discussed, not outcomes for the patient's mother.

The primary difference between a factitious disorder and other somatic disorders is described in which statement? a.Factitious disorders respond well to confrontation as a primary therapeutic technique. b.Factitious disorders have a symptomatology that is actually controlled by the client. c.Factitious disorders have their origins in depression and anxiety. d.Factitious disorders are always self-directed.

b.Factitious disorders have a symptomatology that is actually controlled by the client.

Nico, a 22-year-old patient, is diagnosed with schizophrenia. Which of the following symptoms would alert a provider to a possible diagnosis of schizophrenia in a 22-year-old male client? a.Excessive sleeping with disturbing dreams b.Hearing voices telling him to hurt his roommate c.Withdrawal from college because of failing grades d.Chaotic and dysfunctional relationships with his family and peers

b.Hearing voices telling him to hurt his roommate People diagnosed with schizophrenia all have at least one psychotic symptom such as hallucinations, delusional thinking, or disorganized speech. The other options do not describe schizophrenia but could be caused by a number of problems.

A 38-year-old patient referred for sleep studies reports frequent daytime lethargy, unintended lapses into sleep, and never feeling rested on awakening in the morning. These symptoms support which sleep-related diagnosis? a.Circadian rhythm disorder b.Hypersomnolence c.REM sleep behavior disorder d.Breathing-related sleeping disorder

b.Hypersomnolence The patient with hypersomnolence reports recurrent periods of sleep or unintended lapses into sleep, frequent napping, nonrefreshing nonrestorative sleep regardless of the amount of time slept, and difficulty with full alertness during the wake period. Circadian rhythm sleep disorders occur when there is a misalignment between the timing of the individual's normal circadian rhythm and external factors that affect the timing or duration of sleep. Patients with REM sleep disorder display elaborate motor activity associated with dream mentation. Breathing-related sleep disorder is characterized by frequent upper airway obstruction

Which behavior best supports the diagnosis of posttraumatic stress disorder (PTSD) in a 4-year-old child? a.Overeating b.Hypervigilance c.A drive to be perfect d.Passivity

b.Hypervigilance PTSD in preschool children may manifest as irritability, aggressive or self-destructive behavior, sleep disturbances, problems concentrating, and hypervigilance. None of the other options are characteristic of PTSD in a young child.

A client reveals that she induces vomiting as often as a dozen times a day. The nurse would expect assessment findings to support which electrolyte imbalance? a.Hypernatremia b.Hypokalemia c.Hypercalcemia d.Hypolipidemia

b.Hypokalemia Vomiting causes loss of potassium, leading to hypokalemia. Vomiting is not the trigger for any of the other options presented

When treating impulse control disorders, psychodynamic psychotherapy is directed toward which goal? a.Mastering relaxation techniques b.Identifying the triggers of the rage c.Teaching the client self-distracting techniques d.Helping the client replace the rage with acceptable alternative feelings

b.Identifying the triggers of the rage Psychodynamic psychotherapy focuses on underlying feelings and motivations and explores conscious and unconscious thought processes. In working with impulse control problems, the therapist may help the patient to uncover underlying feelings and reasons behind rage or anger. This may help them to develop better ways to think about and control their behavior. None of the other options are considered goals of this form of therapy

When educating a client diagnosed with bulimia nervosa about the medication fluoxetine, the nurse should include what information about this medication? a.It will reduce the need for cognitive therapy. b.It will be prescribed at a higher than typical dose. c.There are a variety of medications to prescribe if fluoxetine proves to be ineffective. d.Long-term management of symptoms is best achieved with tricyclic antidepressants.

b.It will be prescribed at a higher than typical dose. Research has shown that antidepressant medication together with cognitive-behavioral therapy brings about improvement in bulimic symptoms. Fluoxetine (Prozac), an Selective serotonin reuptake inhibitors (SSRI) antidepressant, has FDA approval for acute and maintenance treatment of bulimia nervosa in adult patients. When fluoxetine is used for bulimia, it is typically at a higher dose than is used for depression. Although no other drugs have FDA approval for this disorder, tricyclic antidepressants helped reduce binge eating and vomiting over short terms.

Empathic listening is therapeutic because it focuses on action? a.Enhancing self-esteem b.Lessening feelings of isolation c.Reducing anxiety d.Encouraging resilience

b.Lessening feelings of isolation Empathic listening can be healing because it can help minimize feelings of isolation. Empathic listening is not focused on any of the other factors suggested by the other options.

Selective inattention is first noted when experiencing which level of anxiety? a.Mild b.Moderate c.Severe d.Panic

b.Moderate When moderate anxiety is present, the individual's perceptual field is reduced and the client is not able to see the entire picture of events.

Disorders that involve sexual behaviors associated with nonhuman objects are identified by which term? a.Pedophilias b.Paraphilias c.Frotteurism d.Sadomasochism

b.Paraphilias The essential features of paraphilias are recurrent and intense sexually arousing fantasies, sexual urges, or behaviors generally involving inanimate objects, the suffering or humiliation of oneself or a partner, or the use of children or other nonconsenting persons.

A nurse assesses a patient diagnosed with dissociative identity disorder. Which finding would likely be part of the patient's history? a.Travel to a foreign country b.Physical or sexual abuse c.Thyroid dysfunction d.Eating disorder

b.Physical or sexual abuse

A slightly obese client reports falling asleep during the daytime even though she has slept all night. Her husband says she snores, and her blood pressure is noted to be in the low hypertensive range. The nurse anticipates that the client will be scheduled for which diagnostic test? a.Hypertension screening b.Polysomnography c.Glycosylated hemoglobin d.Positron emission tomography

b.Polysomnography Polysomnography consists of an electroencephalogram that records respirations, eye movements, and muscle tone during sleep. Sleep apnea becomes apparent if respirations cease frequently during sleep.

You are interviewing Lance, a 31-year-old patient who has been referred to the sexual disorders clinic by his primary care provider. A client describing his problem states, "I can have an orgasm, no problem. It just happens way too soon." This descriptions support what form of sexual dysfunction? a.Erectile disorder b.Premature ejaculation c.Delayed ejaculation d.Male hypoactive sexual desire disorder

b.Premature ejaculation In premature ejaculation, a man persistently or recurrently achieves orgasm and ejaculation before he wishes to. Erectile disorder (also called erectile dysfunction and impotence) refers to failure to obtain and maintain an erection sufficient for sexual activity. In delayed ejaculation, a man achieves ejaculation during coitus only with great difficulty. Male hypoactive sexual desire disorder is characterized by a deficiency or absence of sexual fantasies or desire for sexual activity.

What is the most beneficial nursing intervention directed toward minimizing the discomfort associated with conducting a sexually focused assessment? a.Assure the client that the responses will be kept confidential. b.Provide the client with a rationale for asking the questions. c.Begin with the most relevant, non-personal question. d.Project a relaxed, causal demeanor when questioning the client

b.Provide the client with a rationale for asking the questions. Letting the client know why the questions are being asked increases openness and cooperation

What is the most beneficial nursing intervention directed toward minimizing the discomfort associated with conducting a sexually focused assessment? a.Assure the client that the responses will be kept confidential. b.Provide the client with a rationale for asking the questions. c.Begin with the most relevant, non-personal question. d.Project a relaxed, causal demeanor when questioning the client.

b.Provide the client with a rationale for asking the questions. Letting the client know why the questions are being asked increases openness and cooperation.

What non-habit-forming melatonin receptor agonist is often prescribed for insomnia? a.Zolpidem (Ambien) b.Ramelteon (Rozerem) c.Eszopiclone (Lunesta) d.Zaleplon (Sonata)

b.Ramelteon (Rozerem) Ramelteon (Rozerem) is a short-acting melatonin receptor agonist that has been approved by the FDA for insomnia and is not habit forming

A 20-year-old was sexually molested at age 10, but he can no longer remember the incident. Which ego defense mechanism is in use? a.Projection b.Repression c.Displacement d.Reaction formation

b.Repression Repression is a defense mechanism that excludes unwanted or unpleasant experiences, emotions, or ideas from conscious awareness.

A child who is able to regain mental stability after a traumatic event is said to be demonstrating what trait? a.Autonomy b.Resilience c.Maturity d.Independence

b.Resilience The term resilience refers to positive adaptation, or the ability to maintain or regain mental health despite adversity.

Narcolepsy is a type of a.Amnesia b.Sleep disorder c.Somaticization disorder

b.Sleep disorder

When discussing somatic disorders from a cultural perspective, which statement is true? a.Somatic disorders are rarely observed in males. b.Somatic symptoms vary widely from culture to culture. c.Underdeveloped countries rarely tolerate somatic disorders. d.Secondary gain is seldom a factor in somatic disorders.

b.Somatic symptoms vary widely from culture to culture. The type and frequency of somatic symptoms vary across cultures. Currently, none of the other options have been supported by research.

What is the major reason for the hospitalization of a depressed patient? a.Inability to go to work b.Suicidal ideation c.Loss of appetite d.Psychomotor agitation

b.Suicidal ideation

Which of the following is true of the relationship between bipolar disorder and suicide? a.Patients need to be monitored only in the depressed phase because this is when suicides occur. b.Suicide is a serious risk because nearly 20% of those diagnosed with bipolar disorder commit suicide. c.Patients with bipolar disorder are not considered high risk for suicide. d.As long as patients with bipolar disorder adhere to their medication regimen, there is little risk for suicide.

b.Suicide is a serious risk because nearly 20% of those diagnosed with bipolar disorder commit suicide. Mortality rates for bipolar disorder are severe because substantial numbers of individuals with bipolar disorder will make a suicide attempt at least once in their lifetime. Suicides occur in both the depressed and the manic phase. Bipolar patients are always considered high risk for suicide because of impulsivity while in the manic phase and hopelessness when in the depressed phase. Although staying on medications may decrease risk, there is no evidence to suggest that only patients who stop medications commit suicide.

A student nurse in the emergency department is assigned to care for a client convicted of the sexual abuse of a child. The student is repulsed by the client because of the nature of his crime and doesn't know how to care for the client under these circumstances. What action should the student nurse take? a.Refuse the assignment because personal feelings will prevent the student from providing good care. b.Talk with a faculty member or an experienced nurse in the emergency department. c.Perform the activities of care but not engage in conversation with the client. d.Suggest to the client that he request a different nurse.

b.Talk with a faculty member or an experienced nurse in the emergency department. Nurses may experience distress when providing care for someone who engages in what they view as objectionable, or even reprehensible, acts. This is sometimes compounded by knowing someone who was a victim or having been victimized ourselves. Talking with a faculty member, a nurse mentor, or someone at a mental health clinic can be helpful and important and may even result in better personal understanding and coping. Refusing an assignment is not an option. Performing the activities of care but not engaging in conversation does not appropriately or fully care for the patient. Telling the patient how she feels would be unprofessional and inappropriate, and is putting the burden of our own feelings onto the patient.

Which nursing intervention is designed to help a schizophrenic client minimize the occurrence of a relapse? a.Schedule the client to attend group therapy that includes those who have relapsed. b.Teach the client and family about behaviors associated with relapse. c.Remind the client of the need to return for periodic blood draws to minimize the risk for Relapse. d.Help the client and family adapt to the stigma of chronic mental illness and periodic relapses.

b.Teach the client and family about behaviors associated with relapse. By knowing what behaviors signal impending relapse, interventions can be quickly invoked when the behaviors occur. The earlier the intervention, the greater the likelihood that a recurrence can be averted. None of the other options are effective interventions when considering relapse prevention.

A 72-year-old patient diagnosed with Parkinson's disease is demonstrating behaviors associated with anxiety and has had several falls lately and is reluctant to take medications as prescribed. When his provider orders lorazepam, 1 mg PO bid, the nurse questions the prescription based primarily on what fact? a.The client may become addicted faster than younger patients. b.The client is at risk for falls. c.The client has a history of nonadherence with medications. d.The client should be treated with cognitive therapies because of his advanced age.

b.The client is at risk for falls. An important nursing intervention is to monitor for side effects of the benzodiazepines, including sedation, ataxia, and decreased cognitive function. In a patient who has a history of falls, lorazepam would be contraindicated because it may cause sedation and ataxia leading to more falls. There is no evidence to suggest that elderly patients become addicted faster than younger patients. A history of nonadherence would not lead to you to question this drug order. Medication and other therapies are used congruently with all age levels.

When the nurse asks whether a client is having any thoughts of suicide, the client becomes angry and defensive, shouting, "I'm sick of you people! Are you ever do is ask me the same question over and over. Get out of here!" The nurse's response is based on what fact concerning hostility? a.The client is getting better and is able to be assertive. b.The client may be at high risk for self-harm. c.The client is probably experiencing transference. d.The client may be angry at someone else and projecting that anger to staff.

b.The client may be at high risk for self-harm.

Which statement is true about the characteristics of the oppositional defiant child? a.The defiance is generally directed toward parents and siblings. b.These behaviors are a predictor of future mental health disorders. c.Arguing tends to be more prevalent in boys. d.Girls display more blaming than do boys.

b.These behaviors are a predictor of future mental health disorders. Oppositional defiant disorder is often predictive of emotional disorders in young adulthood. None of the other statements are necessarily correct

Which intervention would be least useful for accurate assessment of the weight of a client diagnosed with anorexia nervosa? a.Weigh 2 times daily first week, then three times weekly. b.Weigh fully clothed before breakfast. c.Do not reweigh client when client requests. d.Permit no oral intake before weighing.

b.Weigh fully clothed before breakfast. Clients should be weighed daily first week, then three times weekly wearing only bra and panties or underwear before ingesting any food or fluids in the morning. Reweighing is not a request that should be afforded to the client.

A client prescribed a monamine oxidase inhibitor (MOA) has a pass to go out to lunch. Given a choice of the following entrees, the client can safely eat a.avocado salad plate. b.fruit and cottage cheese plate. c.kielbasa and sauerkraut. d.liver and onion sandwich.

b.fruit and cottage cheese plate. Fruit and cottage cheese do not contain tyramine. Avocados, fermented food such as sauerkraut, processed meat, and organ meat, contain tyramine. Monoamine oxidase inhibitors inhibit the breakdown of tyramine, which can lead to high blood pressure, a hypertensive crisis, and eventually a cerebrovascular accident. This information makes the other options incorrect.

Which assessment finding in a patient with major depression represents a vegetative sign? a.restlessness b.hypersomnia c.feelings of guilt d.frequent crying

b.hypersomnia

Assessment of a client suspected of experiencing bulimia nervosa calls for the nurse to perform a.a range of motion assessment. b.inspection of body cavities. c.inspection of the oral cavity. d.body fat analysis.

b.inspection of body cavities. Repeated vomiting often causes dental erosions and caries.

A child diagnosed with autism will demonstrate impaired development in which area? a.Adhering to routines b.playing with other children c.swallowing and chewing d.eye-hand coordination

b.playing with other children Autism affects the normal development of the brain in social interaction and communication skills. Symptoms associated with autism spectrum disorders include significant deficits in social relatedness, including communication, nonverbal behavior, and age-appropriate interaction. Other behaviors include stereotypical repetitive speech, obsessive focus on specific objects, over adherence to routines or rituals, hyper- or hypo-reactivity to sensory input, and extreme resistance to change. None of the other options are characteristically associated with autism.

Which nursing diagnosis should be considered for a child with attention deficit hyperactivity disorder ADHD? a.Anxiety b.risk for injury c.defensive coping d.impaired verbal communication

b.risk for injury The child's marked hyperactivity puts him or her at risk for injury from falls, bumping into objects, impulsively operating equipment, pulling heavy objects off shelves, and so forth.

When a hyperactive manic client expresses the intent to strike another client, the initial nursing intervention would be to a.question the client's motive. b.set verbal limits. c.initiate physical confrontation. d.prepare the client for seclusion.

b.set verbal limits. Verbal limit setting should always precede more restrictive measures. Questioning motives does not address the safety issue that exists.

The purpose of the Abnormal Involuntary Movement Scale (AIMS) assessment on a persistently mentally ill client who has been diagnosed with schizophrenia is early detection of a.acute dystonia. b.tardive dyskinesia. c.cholestatic jaundice. d.pseudoparkinsonism.

b.tardive dyskinesia. An AIMS assessment should be performed periodically on clients who are being treated with antipsychotic medication known to cause tardive dyskinesia. This tool is not used to assess or monitor any of the other options.

The nurse is providing care for a patient demonstrating behaviors associated with moderate levels of anxiety. What question should the nurse ask initially when attempting to help the patient deescalate their anxiety? a. "Do you know what will help you manage your anxiety?" b. "Do you need help to manage your anxiety?" c. "Can you identify what was happening when your anxiety began to increase?" d. "Are you feeling anxious right now?"

c. "Can you identify what was happening when your anxiety began to increase?"

Which patient statement supports a diagnosis of narcolepsy? a. "My wife tells me I snore at night." b. "I sleepwalk several nights a week." c. "I have no control over when I fall asleep." d. "My legs feel funny, and that keeps me awake."

c. "I have no control over when I fall asleep."

Malika agrees to try losing weight according to the nurse practitioner's outlined plan. Additional teaching is warranted when Malika states: a. "I am willing to admit I am depressed." b. "Psychotherapy will be a part of my treatment." c. "I prefer to have a gastric bypass rather than use this plan." d. "My comorbid conditions may improve with weight loss."

c. "I prefer to have a gastric bypass rather than use this plan."

Maggie, a child in protective custody, is found to have an imaginary friend, Holly. Her foster family shares this information with the nurse. The nurse teaches the family members about children who have suffered trauma and knows her teaching was effective when the foster mother states: a. "I understand that imaginary friends are abnormal." b. "I understand that imaginary friends are a maladaptive behavior." c. "I understand that imaginary friends are a coping mechanism." d. "I understand that we should tell the child that imaginary friends are unacceptable."

c. "I understand that imaginary friends are a coping mechanism."

Isabel is a straight-A student, yet she suffers from severe test anxiety and seeks medical attention. The nurse interviews Isabel and develops a plan of care. The nurse recognizes effective teaching about mild anxiety when Isabel states: a. "I would like to try a benzodiazepine for my anxiety." b. "If I study harder, my anxiety level will go down." c. "Mild anxiety is okay because it helps me to focus." d. "I have fear that I will fail at college."

c. "Mild anxiety is okay because it helps me to focus."

A homebound patient diagnosed with agoraphobia has been receiving therapy in the home. The nurse evaluates patient teaching is effective when the patient states: a. "I may never leave the house again." b. "Having groceries delivered is very convenient." c. "My risk for agoraphobia is increased by my family history." d. "I will go out again, someday, just not today."

c. "My risk for agoraphobia is increased by my family history."

Claude is a new nurse on the psychiatric unit. He asks a senior nurse on staff for the "best advice" when working with oppositional defiant disorder. Which statement reflects advice on solid therapeutic communication? a. "When correcting behavior, use a loud firm tone." b. "Use language beyond the patient's education level." c. "When setting limits, be specific and outline consequences." d. "An aggressive body language will make the patients respect your position."

c. "When setting limits, be specific and outline consequences."

Which therapeutic communication statement might a psychiatric-mental health registered nurse use when a patient's nursing diagnosis is altered thought processes? a. "I know you say you hear voices, but I cannot hear them." b. "Stop listening to the voices, they are NOT real." c. "You say you hear voices, what are they telling you?" d. "Please tell the voices to leave you alone for now."

c. "You say you hear voices, what are they telling you?"

A 33-year-old female diagnosed with bipolar I disorder has been functioning well on lithium for 11 months. At her most recent checkup, the psychiatric nurse practitioner states, "You are ready to enter the maintenance therapy stage, so at this time I am going to adjust your dosage by prescribing": a. A higher dosage b. Once a week dosing c. A lower dosage d. A different drug

c. A lower dosage

You are caring for Yolanda, a 67-year-old patient who has been receiving hemodialysis for 3 months. Yolanda reports that she feels angry whenever it is time for her dialysis treatment. You attribute this to: a. Organic changes in Yolanda's brain b. A flaw in Yolanda's personality c. A normal response to grief and loss d. Denial of the reality of a poor prognosis

c. A normal response to grief and loss

During a routine health screening, a grieving widow whose husband died 15 months ago reports emptiness, a loss of self, difficulty thinking of the future, and anger at her dead husband. The nurse suggests bereavement counseling. The widow is most likely suffering from: a. Major depression b. Normal grieving c. Adjustment disorder d. Posttraumatic stress disorder

c. Adjustment disorder

Sleep deprivation is considered a safety issue that results in loss of life and property. Psychomotor impairments of sleep deprivation are similar to symptoms caused by: a. Sleeping in excess of 10 hours b. Misuse of caffeine products c. Alcohol consumption d. Working more than 40 hours per week

c. Alcohol consumption

Conversion disorder is described as an absence of a neurological diagnosis that manifests in neurological symptoms. Channeling of emotions, conflicts, and stressors into physical symptoms is thought to be the cause in conversion disorder. Which statement is true? a. People with conversion disorder are extremely upset about often dramatic symptoms. b. Abnormal patterns of cerebral activation have been found in individuals with conversion disorder. c. An organic cause is usually found in most cases of conversion disorder. d. Symptoms can be turned off and on depending on the patient's choice.

c. An organic cause is usually found in most cases of conversion disorder.

Tomas is a 21-year-old male with a recent diagnosis of schizophrenia. Tomas's nurse recognizes that self-medicating with excessive alcohol is common in this disease and can co-occur along with: a. Generally good health despite the mental illness. b. An aversion to drinking fluids. c. Anxiety and depression. d. The ability to express his needs.

c. Anxiety and depression.

The nurse should plan to educate the male patients prescribed a statin medication on the possible development of which commonly observed side effect? a. Impotence b. Gynecomastia c. Decreased libido d. Delayed ejaculation

c. Decreased libido

Phillip, a 63-year-old male, has exposed his genitals in public for all of his adult life, but the act has lost some of the former thrill. A rationale for this change in his experience may be: a. An increasing sense of shame b. Disgust over his lack of control c. Desire waning with age d. Progression into actual assault

c. Desire waning with age

Diane, a 63-year-old mother of three, was brought to the community psychiatric clinic. Diane and her son had a bitter fight over finances. Ever since Diane has been complaining of "a severe pain in my neck." She has seen several doctors who cannot find a physical basis for the pain. The nurse knows that: a. Showing concern for Diane's pain will increase her obsessional thinking. b. Diane's symptoms are manipulative and under conscious control. c. Diane believes there is a physical cause for the pain and will resist a psychological explanation. d. Diane is trying to make her son feel bad about the argument.

c. Diane believes there is a physical cause for the pain and will resist a psychological explanation.

Larry is a 50-year-old man who works about 60 hours per week. He arrives at the clinic seeking assistance with a weight gain of 50 pounds over the past year. Larry admits to sleeping 4 to 5 hours a night. The nurse recognizes that the weight gain may be related to: a. A new onset of diabetes b. Suspected cardiovascular disease c. Dysregulation of hormones that influence appetite d. Comorbidity of depression with obesity

c. Dysregulation of hormones that influence appetite

Ted, a former executive, is now unemployed due to manic episodes at work. He was diagnosed with bipolar I 8 years ago. Ted has a history of IV drug abuse, which resulted in hepatitis C. He is taking his lithium exactly as scheduled, a fact that both Ted's wife and his blood tests confirm. To reduce Ted's mania the psychiatric nurse practitioner recommends: a. Clonazepam (Klonopin) b. Fluoxetine (Prozac) c. Electroconvulsive therapy (ECT) d. Lurasidone (Latuda)

c. Electroconvulsive therapy (ECT)

Which statement accurately describes the effects of emotional trauma on the individual physically? a. Emotional trauma is a distinct category and unrelated to physical problems b. The physical manifestations of emotional trauma are usually temporary c. Emotional trauma is often manifested as physical symptoms d. Patients are more aware of the physical problems caused by trauma

c. Emotional trauma is often manifested as physical symptoms

Which goal should be addressed initially when providing care for 10-year-old Harper who is diagnosed with posttraumatic stress disorder (PTSD)? a. Harper will be able to identify feelings through the use of play therapy. b. Harper and her parents will have access to protective resources available through social services. c. Harper will demonstrate the effective use of relaxation techniques to restore a sense of control over disturbing thoughts. d. Harper and her parents will demonstrate an understanding of the personal human response to traumatic events.

c. Harper will demonstrate the effective use of relaxation techniques to restore a sense of control over disturbing thoughts.

Light projected into the retina is believed to trigger changes in sleep patterns and quality of sleep. Therefore the nurse should suggest: a. Not reading within an hour of bedtime b. Exercising before bedtime in a darkened environment c. Limiting use of electronic devices in the hour before bedtime d. Dimming the screen on cellphones and computers in the evening

c. Limiting use of electronic devices in the hour before bedtime

When Melissa was a small child, she insisted that she was a boy, refused to wear dresses, and wanted to be called Mitch. As Melissa reached puberty, she no longer displayed a desire to be male. This change in identity is considered: a. Gender dysphoria b. Reaction formation c. Normal d. Early transgender syndrome

c. Normal

Child protective services have removed 10-year-old Christopher from his parents' home due to neglect. Christopher reveals to the nurse that he considers the woman next door his "nice" mom, that he loves school, and gets above average grades. The strongest explanation of this response is: a. Temperament b. Genetic factors c. Resilience d. Paradoxical effects of neglect

c. Resilience

Madelyn, a 29-year-old patient recently diagnosed with depression, comes to the mental health clinic complaining of continued difficulty sleeping. One week ago she was started on a selective serotonin reuptake inhibitor (SRRI), fluoxetine (Prozac), for her depressive symptoms. When educating Madelyn your response is guided by the knowledge that: a. SSRIs such as fluoxetine more commonly cause hypersomnolence as opposed to difficulty sleeping. b. The sleep problem is caused by the depression and is unrelated to the medication. c. The neurotransmitters involved in sleep and wakefulness are the same neurotransmitters targeted by many psychiatric medications and the problem may be temporary. d. The medication should be discontinued since sleep is the most important element to her recovery.

c. The neurotransmitters involved in sleep and wakefulness are the same neurotransmitters targeted by many psychiatric medications and the problem may be temporary.

When a nurse uses therapeutic communication with a withdrawn patient who has major depression, an effective method of managing the silence is to: a. Meditate in the quiet environment b. Ask simple questions even if the patient will not answer c. Use the technique of making observations d. Simply sit quietly and leave when the patient falls asleep

c. Use the technique of making observations

A client tells the nurse, "I just don't sleep more than 5 hours at night." The nurse assesses the client best for individual sleep requirements by asking which question? a."Are you aware that some people require less sleep than others?" b."When did this pattern of sleep start for you?" c."Do you usually feel rested and alert when you get up?" d."Are you taking any medication that could affect your sleep?"

c."Do you usually feel rested and alert when you get up?" The most accurate way to determine an individual's sleep requirements is to ask if the person feels fully awake and functions effectively on the sleep he or she is getting. None of the other options address the issue from the client's perspective.

Ali is a 17-year-old patient with bulimia coming to the outpatient mental health clinic for counseling. Which of the following statements by Ali indicates that an appropriate outcome for treatment has been met? a."I purge only once a day now instead of twice." b."I feel a lot calmer lately, just like when I used to eat four or five cheeseburgers." c."I am a hard worker and I am very compassionate toward others." d."I always purge when I'm alone so that I'm not a bad role model for my younger sister."

c."I am a hard worker and I am very compassionate toward others." An appropriate overall goal for the bulimic patient would include that the patient be able to identify personal strengths, leading to improved self-esteem. Purging only once a day instead of two is incorrect because the goal is to refrain from purging altogether. A goal is for the patient to express feelings without food references. Purging when alone is incorrect because the patient is still purging.

Which statement made by a client would support the diagnosis of Illness anxiety disorder? a."I feel confused and disoriented." b."I feel as though I'm outside my body watching what is happening." c."I know I have cancer, but the doctors just cannot find it." d."I woke up one morning, and my left leg was paralyzed from the knee down."

c."I know I have cancer, but the doctors just cannot find it."

A 38-year-old patient is admitted with major depression. Which statement made by the patient alerts the nurse to a common accompaniment to depression? a."I still pray and read my Bible every day." b."My mother wants to move in with me, but I want to independent." c."I still feel bad about my sister dying of cancer. I should have done more for her!" d."I've heard others say that depression is a sign of weakness."

c."I still feel bad about my sister dying of cancer. I should have done more for her!"

A client diagnosed with paranoid schizophrenia refuses food, stating the voices are saying the food is contaminated and deadly. Which response should the nurse provide to this client statement? a."You are safe here in the hospital; nothing bad will happen to you." b."The voices are wrong about the hospital food. It is not contaminated." c."I understand that the voices are very real to you, but I do not hear them." d."Other people are eating the food, and nothing is happening to them."

c."I understand that the voices are very real to you, but I do not hear them." This reply acknowledges the client's reality but offers the nurse's perception that he or she is not experiencing the same thing. This is the only option that provides such support.

A client prescribed fluoxetine demonstrates an understanding of the medication teaching when making which statement? a."I will make sure to get plenty of sunshine and not use sunscreen to avoid a skin reaction." b."I will not take any over-the-counter medication while on the fluoxetine." c."I will report any symptoms of high fever, fast heartbeat, or abdominal pain to my provider right away." d."I will report increased thirst and urination to my provider."

c."I will report any symptoms of high fever, fast heartbeat, or abdominal pain to my provider right away."

Which room placement would be best for a client experiencing a manic episode? a.A shared room with a client with dementia b.A single room near the unit activities area c.A single room near the nurses' station d.A shared room away from the unit entrance

c.A single room near the nurses' station The room placement that provides a nonstimulating environment is best. Nearness to the nurses' station means close supervision can be provided. None of the other options provide low stimulation.

A client newly diagnosed with hypersomnolence asks about what medication will be prescribed to manage this disorder. The nurse will base his/her response based on what knowledge? a.There is no effective medication treatment for hypersomnolence disorder. b.Medication therapy with benzodiazepines may be initiated. c.A stimulant will most likely be prescribed. d.The client will be started on an anticholinesterase inhibitor.

c.A stimulant will most likely be prescribed. Pharmacotherapy with long-acting amphetamine-based stimulants such as methylphenidate and non-amphetamine-based stimulants such as modafinil are helpful in hypersomnolence disorder. The other options are incorrect because there is effective medication treatment; benzodiazepines are sedating and addictive; and anticholinesterase inhibitors are used for the treatment of dementia.

A client, who has been prescribed clozapine 6 weeks ago, reports flulike symptoms including a fever and a very sore throat, the nurse should initiate which nursing intervention? a.Suggest that the client take something for the fever and get extra rest. b.Advise the physician that the client should be admitted to the hospital. c.Arrange for the client to have blood drawn for a white blood cell count. d.Consider recommending a change of antipsychotic medication.

c.Arrange for the client to have blood drawn for a white blood cell count. Antipsychotic medications may cause agranulocytosis, the first manifestation of which may be a sore throat and flulike symptoms.

Assessment for oppositional defiant disorder should include which interventions? a.Assessing the history, frequency, and triggers for violent outbursts b.Assessing moral development, belief system, and spirituality for the ability to understand the impact of hurtful behavior on others, to empathize with others, and to feel remorse c.Assessing issues that result in power struggles and triggers for outbursts d.Assessing sibling birth order to understand the dynamics of family interaction

c.Assessing issues that result in power struggles and triggers for outbursts Oppositional defiant disorder is characterized by defiant behavior, power struggles, outbursts, and arguing with adults, so assessment of these factors would be important. Assessing for violent outbursts refers to assessment for intermittent explosive disorder. Oppositional defiant disorder is not characterized by violent behaviors. Assessing for the ability to understand the impact of hurtful behaviors on others refers to assessment for conduct disorder. Birth order is not known to play a part in oppositional defiant disorder.

A client with paraphilia tendencies tells the nurse that "I'm disgusted with my lifestyle." What is the nurse's initial intervention? a.Assuring the client that this condition responds well to treatment. b.Telling the client that the first step to managing this behavior is recognizing it as unhealthy. c.Assessing the client for the existence of suicidal ideations. d.Recommending inpatient behavioral modification therapy.

c.Assessing the client for the existence of suicidal ideations. Such clients may be severely depressed and have suicidal ideations that must be recognized immediately. The priority intervention is to address client safety.

A child who was physically and sexually abused is at great risk for demonstrating which characteristic? a.Depression b.Suicide attempts c.Bullying and abusing others d.Becoming active in a gang

c.Bullying and abusing others Children who have been abused are at risk for abusing others, as well as for developing dysfunctional patterns in close interpersonal relationships. While the other characteristics may occur, none are as characteristic as the correct option.

The nurse is caring for a patient on day 1 post surgical procedure. The patient becomes visibly anxious and short of breath, and states, "I feel so anxious! Something is wrong!" What action should the nurse take initially in response to the patient's actions? a.Reassure the patient that what they are feeling is normal anxiety and do deep breathing exercises with her. b.Use the call light to inquire whether the patient has been prescribed prn anxiety medication. c.Call for staff help and assess the client's vital signs. d.Reassure the patient that you will stay until the anxiety subsides.

c.Call for staff help and assess the client's vital signs. In anxiety caused by a medical condition, the individual's symptoms of anxiety are a direct physiological result of a medical condition, such as hyperthyroidism, pulmonary embolism, or cardiac dysrhythmias. In this case, Lana is postoperative and could be experiencing a pulmonary embolism, as evidenced by the shortness of breath and anxiety. She needs immediate evaluation for any serious medical condition. The other options would all be appropriate after it has been determined that no serious medical condition is causing the anxiety.

What would be an appropriate expected outcome of the treatment plan for a client diagnosed with a conversion disorder that interferes with the ability to walk effective? a.Client will walk unassisted within 1 week. b.Client will return to a pre-illness level of functioning within 2 weeks.. c.Client will be ableto state two new effective coping skills within 2 weeks. d.Client will assume full self-care within 3 weeks.

c.Client will be ableto state two new effective coping skills within 2 weeks. An appropriate outcome for somatization disorders is to be aware of negative coping strategies and learn new, effective skills for coping within a realistic timeframe. In the other options, the time frames of these outcomes are unrealistic.

The nurse feels uncomfortable talking with a young male client about his sexual problem. Which action should the nurse take? a.Ask another nurse to take over the interview so you don't project your feelings onto the patient. b.Pause the interview and take time to gather your thoughts and do positive self-talk. c.Continue the interview using an appropriate professional tone and matter-of-fact approach. d.Ask Lance whether he would feel more comfortable speaking with a physician about his problem.

c.Continue the interview using an appropriate professional tone and matter-of-fact approach. Remembering your position as a professional and addressing the topics in a tone and manner appropriate of a professional will increase your comfort, along with the patient's. The response in the first option would be confusing to the patient and does not address your feelings or work to resolve them. Pausing the interview would not be appropriate because self-assessment is best done before patient interaction. Asking the patient whether he would feel more comfortable speaking with a physician projects your feelings of being uncomfortable onto the patient and does not carry out your professional role and responsibility.

Which mental health disorder is an example of a somatoform disorder? a.Depersonalization b.Dissociative fugue c.Conversion disorder d.Dissociative identity disorder

c.Conversion disorder Somatic disorders include conversion disorders that are functional neurological disorders. None of the other options are associated with this classification of mental health disorders.

When a child demonstrates a temperament that prompts the mother to say, "She is just so different from me; I just can't seem to connect with her." The nurse should plan to provide which intervention? a.Suggest that the child's father become her primary caregiver. b.Encourage the mother to consider attending parenting classes. c.Counsel the mother regarding ways to better bond with her child. d.Educate the father regarding signs that the child is being physically abused.

c.Counsel the mother regarding ways to better bond with her child. All people have temperaments, and the fit between the child and parent's temperament is critical to the child's development. The caregiver's role in shaping that relationship is of primary importance, and the nurse can intervene to teach parents ways to modify their behaviors to improve the interaction.

Which disorder is characterized by the client's misinterpretation of physical sensations or feelings? a.Somatic disorder b.Factitious disorder c.Illness anxiety disorder d.Conversion disorder

c.Illness anxiety disorder Previously known as hypochondriasis, illness anxiety disorder results in the misinterpretation of physical sensations as evidence of a serious illness. Illness anxiety can be quite obsessive, because thoughts about illness may be intrusive and difficult to dismiss, even when the patient recognizes that his or her fears are unrealistic. This is not an accurate description of any of the other options.

Which statement concerning syndromes seen in other cultures but not seen in our own, such as piblokto, Navajo frenzy witchcraft, and amok should be considered true? a.Dissociative disorders such as dissociative identify disorders b.Physical disorders, not mental disorders c.Culture-bound syndromes that are not dissociative disorders d.Myths, or rumors, because they have not been sufficiently studied to be classified as real.

c.Culture-bound syndromes that are not dissociative disorders Certain culture-bound disorders exist in which there is a high level of activity, a trancelike state, and running or fleeing, followed by exhaustion, sleep, and amnesia regarding the episode. These syndromes, if observed in individuals native to the corresponding geographical areas, should be differentiated from dissociative disorders.

What term is used to identify the condition demonstrated by a person who has numerous hypomanic and dysthymic episodes over a two-year period? a.Bipolar II disorder. b.Bipolar I disorder. c.Cyclothymia. d.Seasonal affective disorder.

c.Cyclothymia. Cyclothymia refers to mood swings involving hypomania and dysthymia of 2 years duration. The mood swings are not severe enough to prompt hospitalization. None of the other options meet that criteria.

Which item of data should be routinely gathered during assessment of a client with a somatoform disorder? a.Potential for violence b.Level of confusion c.Dependence on medication d.Personal identity disturbance

c.Dependence on medication

A Gulf War veteran is entering treatment for post-traumatic stress disorder. What assessment is of importance to this particular client? a.Ascertain how long ago the trauma occurred. b.Find out if the client uses acting-out behavior. c.Determine the use of chemical substances for anxiety relief. d.Establish whether the client has chronic hypertension related to high anxiety.

c.Determine the use of chemical substances for anxiety relief. Substance abuse often coexists with post-traumatic stress disorder.

A manic client tells a nurse "Bud. Crud. Dud. I'm a real stud! You'd like what I have to offer. Let's go to my room." What is the best initial approach to managing this behavior? a.Reprimand the client by stating, "What an offensive thing to suggest!" b.Clarifying the nurse-client relationship by stating, "I don't have sex with clients." c.Distracting the client by suggesting, "It's time to work on your art project." d.Enforcing consequences by responding, "Let's walk down to the seclusion room."

c.Distracting the client by suggesting, "It's time to work on your art project." Distractibility works as the nurse's friend. Rather than discuss the invitation, the nurse may be more effective by redirecting the client. This intervention is both therapeutic and less restrictive.

According to current theory, which statement regarding eating disorders is accurate? a.Eating disorders are psychotic disorders in which patients experience body dysmorphic disorder. b.Eating disorders are frequently misdiagnosed. c.Eating disorders are possibly influenced by sociocultural factors. d.Eating disorders are rarely comorbid with other mental health disorders.

c.Eating disorders are possibly influenced by sociocultural factors. The Western cultural ideal that equates feminine beauty with tall, thin models has received much attention in the media as a cause of eating disorders. Studies have shown that culture influences the development of self-concept and satisfaction with body size. Eating disorders are not psychotic disorders. There is no evidence that eating disorders are frequently misdiagnosed. Comorbidity for patients with eating disorders is more likely than not. Personality disorders, affective disorders, and anxiety frequently occur with eating disorders

Panic attacks in Latin American individuals often involve demonstration of which behavior? a.Repetitive involuntary actions b.Blushing c.Fear of dying d.Offensive verbalizations

c.Fear of dying Panic attacks in Latin Americans and Northern Europeans often involve sensations of choking, smothering, numbness or tingling, as well as fear of dying.

Which of the following symptoms would lead a provider to suspect that a client is experiencing PTSD? Select all that apply. a.Visiting the scene of the accident over and over b.Talking with strangers about the events of the accident c.Flashbacks of the accident d.Hypervigilance e.Irritability f.Difficulty concentrating g.Mania

c.Flashbacks of the accident d.Hypervigilance e.Irritability g.Mania All these symptoms are signs of PTSD.

Which behavior is most indicative of a 4-year-old child diagnosed with Tourette's syndrome? a.Difficulty in social relationships b.Humming while performing activities that require concentration c.Frequent eye blinking d.Difficulty completing tasks on time

c.Frequent eye blinking Persistent motor or vocal ticking is characteristic of Tourette's syndrome. Dysfunctional social relationship is an inconclusive symptom, especially for a 4-year-old. Humming can be a normal response of a child at play. Ineffective time management is usually associated with a child who demonstrates ADHD, not Tourette's syndrome

A client diagnosed with disorganized schizophrenia would have greatest difficulty with the nursing intervention? a.Interacting with a neutral attitude b.Using concrete language c.Giving multistep directions d.Providing nutritional supplements

c.Giving multistep directions The thought processes of the client with disorganized schizophrenia are severely disordered, and severe perceptual problems are present, making it extremely difficult for the client to understand what others are saying. All communication should be simple and concrete and may need to be repeated several times. Ineffective organizational skills would not be a primary factor considering the other options.

A 31-year-old patient admitted with acute mania tells the staff and the other patients that he is on a secret mission for the President of the United States. He states, "I am the only one he trusts, because I am the best!" What term will the nurse use when documenting this behavior? a.Unpredictability b.Rapid cycling c.Grandiosity d.Flight of ideas

c.Grandiosity Grandiosity is inflated self-regard. People with mania may exaggerate their achievements or importance, state that they know famous people, or believe they have great powers. Although patients with mania are unpredictable, the scenario does not describe unpredictability: rapid cycling is switching between mania and depression in a given time period. The scenario does not describe flight of ideas, which means a continuous flow of speech with abrupt topic changes.

The client reveals to the nurse that, "I'm turned on by little girls, not adult women." This statement supports which possible diagnosis? a.Exhibitionism b.Hedonism c.Pedophilia d.Voyeurism

c.Pedophilia Pedophilia involves sexual fantasies, urges, or behaviors with a child aged 13 years or younger.

Which nursing diagnosis should be investigated for clients with somatoform disorders? a.Deficient fluid volume b.Self-care deficit c.Ineffective coping d.Delayed growth and development

c.Ineffective coping Soma is the Greek word for "body," and somatization is the expression of psychological stress through physical symptoms. This information supports that clients generally demonstrate ineffecting coping of anxiety, loneliness, and risk of suicide. None of the other options are associated with somatoform disorders.

The record mentions states that the client habitually relies on rationalization. The nurse might expect the client to present with what behavior? a.Makes jokes to relieve tension. b.Misses appointments. c.Justifies illogical ideas and feelings. d.Behaves in ways that are the opposite of his or her feelings.

c.Justifies illogical ideas and feelings. Rationalization involves justifying illogical or unreasonable ideas or feelings by developing logical explanations that satisfy the teller and the listener.

A woman suddenly finds she cannot see but seems unconcerned about her symptom and tells her husband, "Don't worry, dear. Things will all work out." Her attitude is an example of what process? a.Regression b.Depersonalization c.La belle indifference d.Dissociative amnesia

c.La belle indifference La belle indifference is an attitude of unconcern about a symptom that is unconsciously used to lower anxiety. Such indifference is not observed in any of the other options.

A depressed client tells the nurse, "There is no sense in trying. I am never able to do anything right!" The nurse should identify this cognitive distortion as what response? a.Self-blame b.Catatonia c.Learned helplessness d.Discounting positive attributes

c.Learned helplessness Learned helplessness results in depression when the client feels no control over the outcome of a situation. None of the other options demonstrate these feelings

A client reports insomnia and shares that a friend has recommended a nonprescription hormone product that can be purchased at the local health food store. The nurse suspects that the medication contains which component? a.A benzodiazepine b.A tranquilizer c.Melatonin d.Lithium

c.Melatonin The use of melatonin appears to be helpful in treating sleep problems in the elderly. However, this practice is not without risk. Melatonin products are not approved by the U.S. Food and Drug Administration; therefore, variation may exist in the purity, safety, and effectiveness of the products.

Which problem does NOT fit the group? a.Conversion disorder (functional neurological disorder) b.Illness anxiety disorder (hypochondriasis) c.Munchausen syndrome by proxy d.Somatic symptom disorder

c.Munchausen syndrome by proxy Munchausen by proxy is a factitious disorder. The distracters are somaticization disorders.

Beck's cognitive theory suggests that the etiology of depression is related to what factor? a.Sleep abnormalities b.Serotonin circuit dysfunction c.Negative processing of information d.S belief that one has no control over outcomes

c.Negative processing of information Beck is a cognitive theorist who developed the theory of the cognitive triad of three automatic thoughts responsible for people becoming depressed: (1) a negative, self-deprecating view of oneself; (2) a pessimistic view of the world; and (3) the belief that negative reinforcement will continue. None of the other options are related to this theory.

When the clinician mentions that a client has anhedonia, the nurse can expect that the client will demonstrate what behavior? a.Poor retention of recent events b.A weight loss from anorexia c.No pleasure from previously enjoyed activities d.Difficulty with tasks requiring fine motor skills

c.No pleasure from previously enjoyed activities Anhedonia is the only term that suggests the lack of ability to experience pleasure.

The plan of care for a client who has elaborate washing rituals specifies that response prevention is to be used. Which scenario is an example of response prevention? a.Having the client repeatedly touch "dirty" objects b.Not allowing the client to seek reassurance from staff c.Not allowing the client to wash hands after touching a "dirty" object d.Telling the client that he or she must relax whenever tension mounts

c.Not allowing the client to wash hands after touching a "dirty" object Response prevention is a technique by which the client is prevented from engaging in the compulsive ritual. A form of behavior therapy, response prevention is never undertaken without physician approval

The nurse is planning long-term goals for a 17-year-old male client recently diagnosed with schizophrenia. Which statement should serve as the basis for the goal-setting process? a.If treated quickly following diagnosis, schizophrenia can be cured. b.Schizophrenia can be managed by receiving treatment only at the time of acute exacerbations. c.Patients with schizophrenia often do not fully respond to treatment and have residual symptoms and varying degrees of disability. d.If patients with schizophrenia stay on their drug regimen, they usually lead fully productive lives with no further symptoms.

c.Patients with schizophrenia often do not fully respond to treatment and have residual symptoms and varying degrees of disability Unfortunately, in most cases, schizophrenia does not respond fully to available treatments; it leaves residual symptoms and causes varying degrees of dysfunction or disability. The other options are all untrue of schizophrenia.

A 4 years old is referred to the outpatient mental health clinic after being in a severe car accident during which the child mother died. The father states that the child is withdrawn, not sleeping, having nightmares, and acts out the car accident over and over again when playing. The child states, "It's my fault because I'm bad." What trauma induced disorder does this data support? a.Adjustment disorder b.Dissociative identity disorder c.Posttraumatic stress disorder (PTSD) d.Acute stress disorder (ASD)

c.Posttraumatic stress disorder (PTSD) PTSD in preschool children may manifest as repetitive play that includes aspects of the traumatic event, social withdrawal, and negative emotions such as fear, guilt, anger, horror, sadness, shame, or confusion. Children may blame themselves for the traumatic event and manifest persistent negative thoughts about themselves. Unlike PTSD, adjustment disorder may be diagnosed immediately or within 3 months of exposure. Responses to the stressful event may include combinations of depression, anxiety, and conduct disturbances. Dissociative identity disorder includes the presence of "alters" or other personalities that take over in times of stress. As compared with PTSD that occurs a month after the trauma, ASD occurs from 3 days and up to 1 month after exposure to a highly traumatic event. Individuals with ASD experience three or more dissociative symptoms either during or after the traumatic event, including the following: a sense of numbing, detachment, or absence of emotional responsiveness; a reduction in awareness of surroundings; derealization (a sense of unreality related to the environment); depersonalization (experience of a sense of unreality or self-estrangement); or dissociative amnesia (loss of memory).

A depressed client is noted to pace most of the time, pull at her clothes, and wring her hands. These behaviors are consistent with which term? a.Senile dementia b.Hypertensive crisis c.Psychomotor agitation d.Central serotonin syndrome

c.Psychomotor agitation These behaviors describe the psychomotor agitation sometimes seen in clients with the agitated type of depression. None of the other options are associated so directly with these behaviors

A person who recently gave up smoking and now talks constantly about how smoking fouls the air, causes cancer, and "burns" money that could be better spent to feed the poor is demonstrating which ego defense mechanism? a.Projection b.Rationalization c.Reaction formation d.Undoing

c.Reaction formation Reaction formation keeps unacceptable feelings or behaviors out of awareness by developing the opposite behavior or emotion.

What are the foundational concerns regarding the use of restraint and seclusion when providing care to children? Select all that apply. a.Parents may initiate a lawsuit if injury occurs. b.Staff have conflicted feelings leading to ineffectiveness. c.Research suggests both are psychologically and physically harmful. d.Staff tends to be undertrained in use of restraints in children. e.The principle of least restrictive intervention is a primary concern.

c.Research suggests both are psychologically and physically harmful. e.The principle of least restrictive intervention is a primary concern. Restraint and seclusion have been shown to be psychologically harmful and may also be physically harmful and result in injury or death. To ensure that the civil and legal rights of individuals are maintained, techniques are selected according to the principle of least restrictive intervention. This principle requires that you use more-restrictive interventions only after attempting less restrictive interventions to manage the behavior that have been unsuccessful. The other options are not correct reasons why restraint and seclusion are controversial in children

Which diagnosis from the list below would be given priority for a client diagnosed with bulimia nervosa? a.Disturbed body image b.Chronic low self-esteem c.Risk for injury: electrolyte imbalance d.Ineffective coping: impulsive responses to problems

c.Risk for injury: electrolyte imbalance The client who engages in purging and excessive use of laxatives and enemas is at risk for metabolic acidosis from bicarbonate loss. This electrolyte imbalance is potentially life threatening. While appropriate none of the other options are as likely to risk the client's life.

A teenaged client is being discharged from the psychiatric unit with a prescription for risperidone. The nurse providing medication teaching to the client's mother should provide which response when asked about the risk her son faces for extrapyramidal side effects (EPSs)? a.All antipsychotic medications have an equal chance of producing EPSs. b.Newer antipsychotic medications have a higher risk for EPSs. c.Risperidone is a newer antipsychotic medication and has a lower risk of EPSs than older antipsychotics. d.Advise the mother to ask the provider to change the medication to clozapine instead of risperidone.

c.Risperidone is a newer antipsychotic medication and has a lower risk of EPSs than older antipsychotics. Risperidone is a newer, atypical antipsychotic. All newer antipsychotic medications have a lower incidence of EPSs than older, traditional antipsychotics. The other responses are untrue. There is no reason to advise a medication change at this time.

Assessment of the thought processes of a client diagnosed with depression is most likely to reveal what characteristic? a.Good memory and concentration b.Delusions of persecution c.Self-deprecatory ideation d.Sexual preoccupation

c.Self-deprecatory ideation Depressed clients never feel good about themselves. They have a negative, self-deprecating view of the world. This characteristic is not associated with any of the other options.

Biological theorists suggest that the cause of eating disorders may be related to which factor? a.Normal weight phobia b.Body image disturbance c.Serotonin imbalance d.Dopamine excess

c.Serotonin imbalance The selective serotonin reuptake inhibitors have been shown to improve the rate of weight gain and reduce the occurrence of relapse.

Which statement provides accurate information regarding transvestic disorder? a.Most people with this disorder are homosexual. b.Only men are diagnosed with transvestic disorder. c.Sexual orientation has no bearing on transvestic disorder. d.Transvestic behavior develops in middle adulthood.

c.Sexual orientation has no bearing on transvestic disorder. Unlike in gender dysphorias, in transvestic disorder there are no sexual orientation issues, and people with transvestic disorder do not desire a sex change. Transvestites are usually heterosexual. Although more common in men, women are also diagnosed with transvestic disorder. Transvestic disorder usually develops early in life.

A client asks the nurse to explain what basal sleep requirement is. What is the nurse's best response? a.The basal temperature of your body needed to induce the best sleep. b.The sleep time by your body needed to repair cellular damage. c.The amount of sleep needed to be fully awake and perform well in the daytime. d.The amount of sleep needed to transition to rapid eye movement (REM) sleep.

c.The amount of sleep needed to be fully awake and perform well in the daytime. Basal sleep requirement is the amount of sleep required to feel fully awake and able to sustain normal levels of performance during the periods of wakefulness. The other options do not describe basal sleep requirement.

Dissociative identity disorder is characterized by what event? a.The inability to recall important information b.Sudden, unexpected travel away from home and inability to remember the past c.The existence of two or more subpersonalities, each with its own patterns of thinking d.Recurring feelings of detachment from one's body or mental processes

c.The existence of two or more subpersonalities, each with its own patterns of thinking Dissociation is an unconscious defense mechanism that protects the individual against overwhelming anxiety through an emotional separation. However, this separation results in disturbances in memory, consciousness, self-identity, and perception.

A client diagnosed with post-traumatic stress disorder (PTSD) shows little symptom improvement after being prescribed a selective serotonin reuptake inhibitor (SSRI). The nurse expects that which medication will be prescribed next? a.Beta blocker b.Barbiturate c.Tricyclic antidepressant (TCA) d.Sedative

c.Tricyclic antidepressant (TCA) TCAs or mirtazapine (Remeron) may be prescribed if SSRIs or SNRIs are not tolerated or do not work.

When parents share that their 8-year-old child seems to "always try to be annoying and hateful," the nurse suspects the child is demonstrating which characteristic? a.Emotionally immature b.Anxiety c.Vindictiveness d.Depression

c.Vindictiveness Vindictiveness is defined as spiteful, malicious behavior. The person with this disorder also shows a pattern of deliberately annoying people and blaming others for his or her mistakes or misbehavior. This child may frequently be heard to say "He made me do it!" or "It's not my fault!"

What factors are consistently observed to increase the risk for sleep disturbances? a.Gender and race b.Diet and exercise c.alcohol and tobacco d.income and education

c.alcohol and tobacco There is a strong correlation between alcohol and tobacco use with sleep latency and efficiency problems. None of the other factors are as consistently observed as risk factors

A class of medications commonly prescribed for somatic disorders is a.mood stabilizers. b.antidepressants. c.anxiolytics. d.antipsychotics.

c.anxiolytics. Primary care providers prescribe anxiolytic agents for patients who seem highly anxious and concerned about their symptoms. Individuals experiencing many somatic complaints often become dependent on medication to relieve pain or anxiety or to induce sleep.

A client frantically reports to the nurse that "You have got to help me! Something terrible is happening. I can't think. My heart is pounding, and my head is throbbing." The nurse should assess the client's level of anxiety as a.mild. b.moderate. c.severe. d.panic.

c.severe. Severe anxiety is characterized by feelings of falling apart and impending doom, impaired cognition, and severe somatic symptoms such as headache and pounding heart. Mild and moderate levels of anxiety do not demonstrate these feels while panic is even more intense than the scenario implies.

Hormone therapy for the purpose of surgical gender reassignment is initiated when the client has a.successfully demonstrated a genuine intent to change genders. b.taken on the dress and manners of the preferred gender. c.successfully lived the crossgender role in all aspects of life. d.taken all legal steps to change name and legal status.

c.successfully lived the crossgender role in all aspects of life. After living as a member of the desired gender, if the client still wishes to proceed with gender reassignment, hormone therapy can be initiated.

When prescribed lorazepam (Ativan) 1 mg po qid for 1 week for generalized anxiety disorder, the nurse should a.question the physician's order because the dose is excessive. b.explain the long-term nature of benzodiazepine therapy. c.teach the client to limit caffeine intake. d.tell the client to expect mild insomnia.

c.teach the client to limit caffeine intake. Caffeine is an antagonist of antianxiety medication.

A client hospitalized for a psychotic relapse is being discharged home to family. Which topic is important to address when teaching both the patient and the family to recognize possible signs of impending mania? a-Increased appetite b-Decreased social interaction c-Increased attention to bodily functions d-Decreased sleep

d-Decreased sleep Changes in sleep patterns are especially important because they usually precede mania. Even a single night of unexplainable sleep loss can be taken as an early warning of impending mania. The other options do not indicate impending mania.

Which response by a 15-year-old demonstrates a common symptom observed in patients diagnosed with major depressive disorder? a. "I'm so restless. I can't seem to sit still." b. "I spend most of my time studying. I have to get into a good college." c. "I'm not trying to diet, but I've lost about 5 pounds in the past 5 months." d. "I go to sleep around 11 p.m. but I'm always up by 3 a.m. and can't go back to sleep."

d. "I go to sleep around 11 p.m. but I'm always up by 3 a.m. and can't go back to sleep."

Many people allow life circumstances to dictate their amount of sleep instead of recognizing sleep as a priority. Which statement will the nurse recognize as progress in the patient's sleep hygiene program? a. "I go to bed even if I am not sleepy, hoping I will fall asleep." b. "I have one glass of red wine at bedtime each night." c. "I take a nap each day to 'catch up' on my sleep deficit." d. "I have removed the television from my bedroom."

d. "I have removed the television from my bedroom."

Which patient statement suggests a concern over one's ability to perform sexually? a. "My partner and I aren't as close as we once were." b. "I'm not as desirable as I once was." c. "My personal life has changed a lot." d. "I'm not the partner I used to be."

d. "I'm not the partner I used to be."

An incest survivor undergoing treatment at the mental health clinic is relieved when she learns that her anxiety and depression are: a. Going to be eradicated with treatment b. Normal and will soon pass c. Abnormal but will pass d. A normal reaction to posttraumatic events

d. A normal reaction to posttraumatic events

Which patient is at greatest risk for developing a stress- induced myocardial infarction? a. A patient who lost a child in an accidental shooting 24 hours ago b. A woman who has begun experiencing early signs of menopause c. A patient who has spent years trying to sustain a successful business d. A patient who was diagnosed with chronic depression 10 years ago

d. A patient who was diagnosed with chronic depression 10 years ago

The school nurse has been alerted to the fact that an 8-year-old boy routinely playacts as a police officer "locking up" other children on the playground to the point where the children get scared. The nurse recognizes that this behavior is most likely an indication of: a. The need to dominate others b. Inventing traumatic events c. A need to develop close relationships d. A potential symptom of traumatization

d. A potential symptom of traumatization

You are caring for Aaron, a 38-year-old patient diagnosed with somatic symptom disorder. When interacting with you, Aaron continues to focus on his severe headaches. In planning care for Aaron, which of the following interventions would be appropriate? a. Call for a family meeting with Aaron in attendance to confront Aaron regarding his diagnosis. b. Educate Aaron on alternative therapies to deal with pain. c. Improve reality testing by telling Aaron that you do not believe that the headaches are real. d. After a limited discussion of physical concerns, shift focus to feelings and effective coping skills.

d. After a limited discussion of physical concerns, shift focus to feelings and effective coping skills.

Which intervention will promote independence in a patient being treated for bulimia nervosa? a. Have the patient monitor daily caloric intake and intake and output of fluids. b. Encourage the patient to use behavior modification techniques to promote weight gain behaviors. c. Ask the patient to use a daily log to record feelings and circumstances related to urges to purge. d. Allow the patient to make limited choices about eating and exercise as weight gain progresses.

d. Allow the patient to make limited choices about eating and exercise as weight gain progresses.

The nurse is providing medication education to a patient who has been prescribed lithium to stabilize mood. Which early signs and symptoms of toxicity should the nurse stress to the patient? Select all that apply. a. Increased attentiveness b. Getting up at night to urinate c. Improved vision d. An upset stomach for no apparent reason e. Shaky hands that make holding a cup difficult

d. An upset stomach for no apparent reason e. Shaky hands that make holding a cup difficult

To maximize the therapeutic effect, which lifestyle practice should the nurse discourage for a patient who has been recently prescribed an antianxiety medication? a. Eating high protein foods. b. Using acetaminophen without first discussing it with a healthcare provider c. Taking medications after eating dinner or while having a bedtime snack d. Buying a large coffee with sugar and extra cream each morning on the way to work

d. Buying a large coffee with sugar and extra cream each morning on the way to work

Luc's family comes home one evening to find him extremely agitated and they suspect in a full manic episode. The family calls emergency medical services. While one medic is talking with Luc and his family, the other medic is counting something on his desk. What is the medic most likely counting? a. Hypodermic needles b. Fast food wrappers c. Empty soda cans d. Energy drink containers

d. Energy drink containers

Larry, a middle-aged male in a treatment facility, is loudly displaying anger in the day room with a visiting family member. It is obvious to the nurse this pattern has played out before. Violence is often escalated when family members or authority figures: a. Use a soft tone of voice to gain control of the situation b. Move away from the agitated person in fear c. Use simple words to communicate d. Engage in a power struggle

d. Engage in a power struggle

What precipitating emotional factor has been associated with an increased incidence of cancers? Select all that apply. a. Anxiety b. Job-related stress c. Acute grief d. Feelings of hopelessness and despair from depression e. Prolonged, intense stress

d. Feelings of hopelessness and despair from depression e. Prolonged, intense stress

Living comfortable and materialistic lives in Western societies seems to have altered the original hierarchy proposed by Maslow in that: a. Once lower level needs are satisfied, no further growth feels necessary b. Self-actualization is easier to achieve with financial stability c. Esteem is more highly valued than safety d. Focusing on materialism reduces interests in love, belonging, and family

d. Focusing on materialism reduces interests in love, belonging, and family

Cabot has multiple symptoms of depression including mood reactivity, social phobia, anxiety, and overeating. With a history of mild hypertension, which classification of antidepressants dispensed as a transdermal patch would be a safe medication? a. Tricyclic antidepressants b. Selective serotonin reuptake inhibitors c. Serotonin and norepinephrine reuptake inhibitors d. Monoamine oxidase inhibitor

d. Monoamine oxidase inhibitor

Which characteristic presents the greatest risk for injury to others by the patient diagnosed with schizophrenia? a. Depersonalization b. Pressured speech c. Negative symptoms d. Paranoia

d. Paranoia

A female patient diagnosed with schizophrenia has been prescribed a first-generation antipsychotic medication. What information should the nurse provide to the patient regarding her signs and symptoms? a. Her memory problems will likely decrease. b. Depressive episodes should be less severe. c. She will probably enjoy social interactions more. d. She should experience a reduction in hallucinations.

d. She should experience a reduction in hallucinations.

Lucas is a nurse on a medical floor caring for Kelly, a 48-year-old patient with newly diagnosed type 2 diabetes. He realizes that depression is a complicating factor in the patient's adjustment to her new diagnosis. What problem has the most potential to arise? a. Development of agoraphobia b. Treatment nonadherence c. Frequent hypoglycemic reactions d. Sleeping rather than checking blood sugar

d. Sleeping rather than checking blood sugar

Which activity is most appropriate for a child with ADHD? a. Reading an adventure novel b. Monopoly c. Checkers d. Tennis

d. Tennis

When patients diagnosed with schizophrenia suffer from anosognosia, they often refuse medication, believing that: a. Medications provided are ineffective. b. Nurses are trying to control their minds. c. The medications will make them sick. d. They are not actually ill.

d. They are not actually ill.

Gilbert, age 19, is described by his parents as a "moody child" with an onset of odd behavior about at age 14, which caused Gilbert to suffer academically and socially. Gilbert has lost the ability to complete household chores, is reluctant to leave the house, and is obsessed with the locks on the windows and doors. Due to Gilbert's early and slow onset of what is now recognized as schizophrenia, his prognosis is considered: a. Favorable with medication b. In the relapse stage c. Improvable with psychosocial interventions d. To have a less positive outcome

d. To have a less positive outcome

Obtaining a sexual history can be embarrassing for the patient and practitioner. Experience with addressing the topic can help, as well as: a. Using informal language familiar to the patient's age b. Avoiding specifics and keeping the interview on general topics c. Avoiding eye contact d. Using a professional tone of voice and a relaxed posture

d. Using a professional tone of voice and a relaxed posture

Emily asks you what kind of therapy will help her. Based on current knowledge, what form of therapy is most appropriate for a client diagnosed with a conversion disorder? a."A combination of antianxiety and antidepressant therapy is the most effective therapy." b."Aversion therapy is often used because in effect you are punishing yourself by not being able to walk." c."Modeling will be used; as you see desired behaviors modeled by the therapist you will be able to also achieve the expected outcome." d."Cognitive-behavioral therapy (CBT) has been shown to consistently provide the best outcome for these types of disorders."

d."Cognitive-behavioral therapy (CBT) has been shown to consistently provide the best outcome for these types of disorders." CBT is the most consistently supported treatment for the full spectrum of somatic disorders. All the other options are incorrect and do not describe the most used and effective therapy for this disorder.

The nurse appropriately assesses an obese, hypertensive, Type 2 diabetic client when asking: a."Do you regularly have nightmares?" b."Is getting to sleep a problem for you?" c."Do you snooze when you sleep?" d."How much sleep do you usually get each night?"

d."How much sleep do you usually get each night?" Short sleep duration has been associated with obesity, cardiovascular disease, hypertension, and diabetes. Neither nightmares nor snoring are as directly associated with short sleep duration.

What statement by a client would indicate that goals for treatment for a somatization disorder are being achieved? a."I feel less anxiety than before." b."My memory is better than it was a month ago." c."I take my medications just as the physician prescribed." d."I don't think about my symptoms all the time as I used to."

d."I don't think about my symptoms all the time as I used to."

A client diagnosed with schizophrenia states to the nurse, "My, oh my. My mother is brother. Anytime now it can happen to my mother." Your best response would be: a."You are having problems with your speech. You need to try harder to be clear." b."You are confused. I will take you to your room to rest a while." c."I will get you a prn medication for agitation." d."I'm sorry, I didn't understand that. Do you want to talk more about your mother as we did yesterday?"

d."I'm sorry, I didn't understand that. Do you want to talk more about your mother as we did yesterday?" The guidelines that are useful in communicating with a patient with disorganized or bizarre speech are to place the difficulty in understanding on yourself, not the patient, and look for themes that may be helpful in interpreting what the patient wants to say. Telling the patient he needs to try harder to be clearer is unrealistic since the patient would be unable do this. The other options are not useful in communicating with this patient and attempting to find common themes.

A symptom commonly associated with panic attacks? a.Obsessions b.Apathy c.Fever d.Fear of impending doom

d.Fear of impending doom The feelings of terror present during a panic attack are so severe that normal function is suspended, the perceptual field is severely limited, and misinterpretation of reality may occur.

When a client diagnosed with paranoid schizophrenia tells the nurse, "I have to get away. The volmers are coming to execute me," an appropriate response for the nurse would be: a."You are safe here. This is a locked unit, and no one can get in." b."I do not believe I understand the word volmers. Tell me more about them." c."Why do you think someone or something is going to harm you?" d."It must be frightening to think something is going to harm you."

d."It must be frightening to think something is going to harm you." This response focuses on the client's feelings and neither directly supports the delusion nor denies the client's experience. Option A gives global reassurance. Option B encourages elaboration about the delusion. Option C asks for information that the client will likely be unable to answer.

An adolescent with a conduct disorder is taken to the principal's office after assaulting and injuring another student. Which comment is this adolescent most likely to make? a."I lost my temper, but it will not happen again." b."I'm sorry and embarrassed that this happened." c."I failed my math test, and guess I was just having a bad day." d."So what if that kid was hurt? I should have beaten him more."

d."So what if that kid was hurt? I should have beaten him more."

A client diagnosed with a chronic sleep disorders begins to cry and states, "I can't keep going like this! I work in a bank and if I can't function correctly I'll lose my job. I just don't think I'll get better." Which statement provides a therapeutic response to the client's concerns? a."Don't worry! I'm sure with treatment everything will get better." b."You are not alone. Many people who come for sleep studies are going through the same thing." c."You seem so sad. May I ask if something else is troubling you?" d."There is much hope for improvement through treatment. Let's talk about some strategies for your problems at work."

d."There is much hope for improvement through treatment. Let's talk about some strategies for your problems at work." This response instills hope regarding the ability of the patient to improve and suggests positive strategies for daily functioning.

What is the usual time period that medications to treat insomnia are usually prescribed? a.1 to 2 days b.1 to 2 months c.3 weeks d.2 weeks

d.2 weeks Nurses frequently provide education about the benefits of a particular sleep medication, the side effects, untoward effects, and the fact that medications are usually prescribed for no longer than 2 weeks because tolerance and withdrawal may result.

Which child is at greatest risk for developing attachment problems as a result of a neurobiological development? a.A 13-year-old male b.A 10-year-old female c.A 7-year-old male d.A 4-year-old female

d.A 4-year-old female The developing brain is particularly vulnerable to adverse events because the most rapid brain development occurs in the first five years of life. The right hemisphere is involved in processing social-emotional information, promoting attachment functions, regulating body functions, and in supporting the individual in survival and in coping with stress. Since the right brain develops first and is involved with developing templates for relationships and regulation of emotion and bodily function, early attachment relationships are particularly important for healthy development and life-long health.

The nurse caring for a client experiencing a panic attack anticipates that the psychiatrist would order a stat dose of which classification of medications? a.Standard antipsychotic medication. b.Tricyclic antidepressant medication. c.Anticholinergic medication. d.A short-acting benzodiazepine medication.

d.A short-acting benzodiazepine medication. A short-acting benzodiazepine is the only type of medication listed that would lessen the client's symptoms of anxiety within a few minutes. Anticholinergics do not lower anxiety; tricyclic antidepressants have very little antianxiety effect and have a slow onset of action; and a standard antipsychotic medication will lower anxiety but has a slower onset of action and the potential for more side effects.

A client admitted with major depression and suicidal ideation with a plan to overdose is preparing for discharge and asks you, "Why did I get a prescription for only 7 days of amitriptyline?" The nurse's response is based on what fact? a.Amitriptyline is very expensive, so the patient may have to buy fewer at a time. b.The goal is to see how the client responds to the first week of medication to evaluate its effectiveness. c.The health care provider wants to see whether any side effects occur within the first week of administration. d.Amitriptyline is lethal in overdose.

d.Amitriptyline is lethal in overdose.

Dysthymia cannot be diagnosed unless it has existed for what period of time? a.At least 3 months b.At least 6 months c.At least 1 year d.At least 2 years

d.At least 2 years Dysthymia is a chronic condition that by definition has to have existed for longer than 2 years. None of the other options present a sufficient time period

Which behavior consistently demonstrated by a child is a predictor of future antisocial personality disorder in adults? a.Sadness b.Remorse c.Guilt d.Callousness

d.Callousness

Studies have shown a correlation between mental disorders and which medical condition? a.Psoriasis b.Asthma c.Chronic renal failure d.Cardiovascular disease

d.Cardiovascular disease Studies in recent years have contributed to the growing body of evidence indicating a link between mental disorders and medical conditions such as cardiovascular disease and cancer. No such correlation has been proven between medical and any suggest psychiatric conditions.

Which statement about somatoform disorders is true? a.An organic basis exists for each group of disorders. b.Nurses perceive clients with these disorders as easy to care for. c.No relation exists between these disorders and early childhood loss or trauma. d.Clients lack awareness of the relations among symptoms, anxiety, and conflicts.

d.Clients lack awareness of the relations among symptoms, anxiety, and conflicts.

Working to help the client view an occurrence in a more positive light is referred to by which term? a.Flooding b.Desensitization c.Response prevention d.Cognitive restructuring

d.Cognitive restructuring The purpose of cognitive restructuring is to change the individual's negative view of an event or a situation to a view that remains consistent with the facts but that is more positive.

It is most important for the nurse to employ which holistic strategy when managing clients diagnosed with a somatization disorder? a.Utilizing many different therapeutic strategies or modalities for enhanced coping b.Involving every member of the family as well as the patient in treatment c.Incorporating spirituality and religion into treatment d.Considering all dimensions of the patient, including biological, psychological, and sociocultural

d.Considering all dimensions of the patient, including biological, psychological, and sociocultural It is important to use a holistic approach in nursing care so that we may address the multidimensional interplay of biological, psychological, and sociocultural needs and its effects on the somatization process. All nurses need to be aware of the influence of environment, stress, individual lifestyle, and coping skills of each patient. The other options do not explain the concept of holistic care.

Schizophrenia is best characterized as presenting which personality trait? a.Split b.Multiple c.Ambivalent d.Deteriorating

d.Deteriorating The course of schizophrenia is marked by recurrent acute exacerbations. With each relapse of psychosis, an increase in residual dysfunction and deterioration occurs.

The nurse is providing teaching to a preoperative patient just before surgery. The patient is becoming more and more anxious and begins to report dizziness and heart pounding. The patient also appears confused and is trembling noticeably. Considering the scenario, what decision should the nurse make? a.To reinforce the preoperative teaching by restating it slowly. b.Have the patient read the teaching materials instead of providing verbal instruction. c.Have a family member read the preoperative materials to the patient. d.Do not attempt any further teaching at this time.

d.Do not attempt any further teaching at this time. Patients experiencing severe anxiety, as the symptoms suggest, are unable to learn or solve problems. The other options would not be effective because you are still attempting to teach someone who has a severe level of anxiety

Which medication is FDA approved for treatment of anxiety in children? a.Sertraline b.Fluoxetine c.Clomipramine d.Duloxetine

d.Duloxetine A few drugs are approved specifically for anxiety and obsessive-compulsive disorders in children and adolescents. The FDA approved the selective SNRI duloxetine (Cymbalta) in 2014 for children aged 7 to 17 years for generalized anxiety disorder. The FDA has approved four medications for use in children with obsessive-compulsive disorder. They are clomipramine (Anafranil), fluoxetine (Prozac), fluvoxamine (Luvox), and sertraline (Zoloft).

A client prescribed a selective serotonin reuptake inhibitor mentions taking the medication along with the St. John's wort daily. The nurse should provide the client with what information regarding this practice? a.Agreeing that this will help the client to remember the medications. b.Caution the client to drink several glasses of water daily. c.Suggest that the client also use a sun lamp daily. d.Explain the high possibility of an adverse reaction.

d.Explain the high possibility of an adverse reaction. Serotonin malignant syndrome is a possibility if St. John's wort is taken with other antidepressants. None of the other options are relevant to the situation.

What is the major distinction between fear and anxiety? a.Fear is a universal experience; anxiety is neurotic. b.Fear enables constructive action; anxiety is dysfunctional. c.Fear is a psychological experience; anxiety is a physiological experience. d.Fear is a response to a specific danger; anxiety is a response to an unknown danger.

d.Fear is a response to a specific danger; anxiety is a response to an unknown danger. Fear is a response to an objective danger; anxiety is a response to a subjective danger.

Which subjective symptom should the nurse would expect to note during assessment of a client diagnosed with anorexia nervosa? a.Lanugo b.Hypotension c.25-lb weight loss d.Fear of gaining weight

d.Fear of gaining weight Fear of weight gain is the only subjective data listed, and it is universally true of clients diagnosed with anorexia nervosa.

A bipolar client tells the nurse, "I have the finest tenor voice in the world. The three tenors who do all those TV concerts are going to retire because they can't compete with me." What term should the nurse use to identify this behavior? a.Flight of ideas b.Distractibility c.Limit testing d.Grandiosity

d.Grandiosity Exaggerated belief in one's own importance, identity, or capabilities is seen with grandiosity. None of the other options are associated with this behavior.

The nurse can determine that inpatient treatment for a client diagnosed with an eating disorder would be warranted when which assessment data is observed? a.Weighs 10% below ideal body weight. b.Has serum potassium level of 3 mEq/L or greater. c.Has a heart rate less than 60 beats/min. d.Has systolic blood pressure less than 70 mm Hg.

d.Has systolic blood pressure less than 70 mm Hg. Systolic blood pressure of less than 70 mm Hg is one of the established criteria signaling the need for hospitalization of a client with anorexia nervosa. It suggests severe cardiovascular compromise.

A client who is 16 years old, 5 foot, 3 inches tall, and weighs 80 pounds eats one tiny meal daily and engages in a rigorous exercise program. Which nursing diagnosis addresses this assessment data? a.Death anxiety b.Ineffective denial c.Disturbed sensory perception d.Imbalanced nutrition: less than body requirements

d.Imbalanced nutrition: less than body requirements A body weight of 80 pounds for a 16-year-old who is 5 foot, 3 inches tall is ample evidence of this diagnosis.

Delusionary thinking is a characteristic of which form of anxiety? a.Chronic anxiety b.Acute anxiety c.Severe anxiety d.Panic level anxiety

d.Panic level anxiety Panic level anxiety is the most extreme level and results in markedly disturbed thinking greater than in any of the other options.

An older adult client is reporting symptomatology that suggests REM sleep behavior disorder (RSBD). Which comorbid condition should the nurse assess for? a.Lymphoma b.Hypertension c.Acute renal failure d.Parkinson's disease

d.Parkinson's disease RSBD is seen in elderly males as they begin to develop neurological pathologies such as Parkinson's disease.

Hypoalbuminemia in a patient with an eating disorder would produce which assessment finding? a.Lanugo b.Jaundice c.Amenorrhea d.Peripheral edema

d.Peripheral edema

When a client experiences four or more mood episodes in a 12-month period, which term is used to describe this behavior? a.Dyssynchronous b.Incongruent c.Cyclothymic d.Rapid cycling

d.Rapid cycling Rapid cycling implies four or more mood episodes in a 12-month period, as well as more severe symptomatology. None of the other options are associated with this characteristic behavior.

A client has reached the stable plateau phase of schizophrenia. What is the appropriate clinical planning focus for this client? a.Safety and crisis intervention b.Acute symptom stabilization c.Stress and vulnerability assessment d.Social, vocational, and self-care skills

d.Social, vocational, and self-care skills During the stable plateau phase of schizophrenia, planning is geared toward client and family education and skills training that will help maintain the optimal functioning of schizophrenic individuals in the community. All the other options should have been handled previously

Tara and Aaron are twins who are both diagnosed with schizophrenia. Aaron was diagnosed at 23 years old and Tara at 31 years old. Based on your knowledge of early and late onset of schizophrenia, which of the following is true? a.Tara and Aaron have the same expectation of a poor long-term prognosis. b.Tara will experience more positive signs of schizophrenia such as hallucinations. c.Aaron will be more likely to hold a job and live a productive life. d.Tara has a better chance for positive outcomes because of later onset.

d.Tara has a better chance for positive outcomes because of later onset. Female patients diagnosed with schizophrenia between the ages of 25 and 35 years have better outcomes than do their male counterparts diagnosed earlier. These two patients do not have the same expectation of a poor prognosis. There is no evidence suggesting that Tara will have more positive signs of schizophrenia. It is actually more unlikely that Aaron will be able to live a productive life because of his earlier onset, which has a poorer prognosis

Which side effect of antipsychotic medication is generally nonreversible? a.Anticholinergic effects b.Pseudoparkinsonism c.Dystonic reaction d.Tardive dyskinesia

d.Tardive dyskinesia Tardive dyskinesia is not always reversible with discontinuation of the medication and has no proven cure. The other side effects often appear early in therapy and can be minimized with treatment.

Parents express concern when their 5-year-old child, who is receiving treatment for cancer, keeps referring to an imaginary friend, Candy. Which response should the nurse provide to best address the parent's concerns? a.Children of this age usually have imaginary friends. b.It is nothing to worry about unless the child starts to socially isolate. c.The child needs more of their one-on-one attention. d.The imaginary friend is a coping mechanism the child is using.

d.The imaginary friend is a coping mechanism the child is using. Often traumatized children feel responsible for what happened to them and are frightened by flashbacks, amnesia, or hallucinations that may be due to trauma. For example, a child may use imaginary friends as a coping mechanism. This option addresses the parents' concern most effectively.

When the wife of a manic client asks about genetic transmission of bipolar disorder, the nurse's answer should be predicated on which information? a.No research exists to suggest genetic transmission. b.Much depends on the socioeconomic class of the individuals. c.Highly creative people tend toward development of the disorder. d. The rate of bipolar disorder is higher in relatives of people with bipolar disorder

d.The rate of bipolar disorder is higher in relatives of people with bipolar disorder. This understanding will allow the nurse to directly address the question. Responses based on the other statements would be tangential or untrue.

A 10-year-old who is frequently disruptive in the classroom begins to fidget and then moves on to disruptive behavior. What is the most appropriate initial technique for managing this sort of disruptive behavior? a.Therapeutic holding b.Seclusion c.Quiet room d.Touch control

d.Touch control The appropriate adult can move closer to the child and place a hand on his/her arm or an arm around his/her shoulder for a calming effect when the fidgeting is first noted. The closeness signals the child to use self-control. It is the least restrictive treatment approach and should be tried initially; before any of the other options

A client explains that he is heterosexual but prefers to dress in feminine clothing. This characteristic behavior is suggestive of which sexual disorder? a.Fetishism b.Exhibitionism c.Voyeurism d.Transvestism

d.Transvestism Transvestism is a paraphilia that involves dressing in the clothing of the opposite sex. This behavior is not characteristic of any of the other options.

Which medication for insomnia is classified as an antidepressant rather than a hypnotic? a.Zaleplon (Sonata) b.Zolpidem (Ambien) c.Triazolam (Halcion) d.Trazodone (Desyrel)

d.Trazodone (Desyrel) Trazodone is an antidepressant medication that is often used to treat insomnia. It does not cause dependency.

A parent is shopping with a 5-year-old child in a large, busy urban mall. The parent suddenly realizes the child is missing. Which level of anxiety? a.mild b.moderate c.severe d.panic

d.panic

Loose associations in a person with schizophrenia indicate: a.paranoia b.mood instability c.depersonalization d.poorly organized thinking

d.poorly organized thinking

A client, prescribed which class of antidepressantive medication should be monitored for the development of premature ejaculation? a.Monoamine oxidase (MAO) inhibitors b.Tricyclic antidepressants c.Atypical antipsychotics d.selective serotonin reuptake inhibitor (SSRI) antidepressants

d.selective serotonin reuptake inhibitor (SSRI) antidepressants Treatments include antidepressants in the SSRI category. Conversely, pharmacotherapy may cause erectile dysfunction, and medications may need to be evaluated for change or dose reduction. The other options are not used for premature ejaculation.

A poorly developed sense of empathy is thought to be the result of having what life experience? a.A family history of mental illness b.a low serum testosterone level c.suffered head trauma at an early age d.unmet physical and emotional needs

d.unmet physical and emotional needs

As a nurse assesses a new client, the nurse makes sure the door remains open. Which type of communication factor is this action? 1-enviornmental 2-relationship 3-personal

1-environmental environmental factors that may affect the interview include physical (background noise, lack of privacy, uncomfortable accommodations) and societal (sociopolitical, historical, economical, presence and expectations of others)

A nurse assesses a newly hospitalized client. Which communication will the nurse use most? 1-rapid, high-pitch voice tone 2-closed-ended questions 3-direct eye contact 4-frequent touch

2-closed-ended questions close-ended questions are the most useful in initial assessment or ascertain information

A cognitive therapist would help a client restructure the thought "I am stupid!" to which statement? A-"What I did was stupid." B-"I am not as smart as others." C-"Things usually go wrong for me." D-"Things like this should not happen to anyone."

A-"What I did was stupid." Cognitive therapists help clients identify, reality test, and correct distorted conceptualizations and dysfunctional beliefs, such as realizing that doing a stupid thing does not mean the person is stupid.

What is the premise underlying behavioral therapy? A-Behavior is learned and can be modified. B-Behavior is a product of unconscious drives. C-Motives must change before behavior changes. D-Behavior is determined by cognitions; change in cognitions produces new behavior.

A-Behavior is learned and can be modified. The premise underlying behavior therapy is that behavior is learned and can be modified. Behaviorists agree that behavior can be changed without insight into the underlying cause.

The nurse is finding it difficult to provide structure and set limits for a client. The nurse should self-evaluate for which characteristic of a dysfunctional nurse-patient relationship? A-Boundary blurring B-Value dissonance C-Covert anger D-Empathy

A-Boundary blurring Boundary blurring is often signaled by the nurse being either too helpful or not helpful enough.

The nurse would address which of the following goals in attempting to establish a therapeutic nurse-client relationship?Select all that apply. A-Helping patients examine self-defeating behaviors and test alternatives B-Promoting self-care and independence C-Providing the client with opportunities to socialize D-Assisting patients with problem solving to help facilitate activities of daily living E-Facilitating communication of distressing thoughts and feelings

A-Helping patients examine self-defeating behaviors and test alternatives B-Promoting self-care and independence D-Assisting patients with problem solving to help facilitate activities of daily living E-Facilitating communication of distressing thoughts and feelings

Which of the following statements are true regarding the differences between a social relationship and a therapeutic relationship? Select all that apply. A-In a social relationship, both parties' needs are met; in a therapeutic relationship, only the patient's needs are to be considered. B-A social relationship is instituted for the main purpose of exploring one member's feelings and issues; a therapeutic relationship is instituted for the purpose of friendship. C-Giving advice is done in social relationships; in therapeutic relationships giving advice is not usually therapeutic. D-In a social relationship, both parties come up with solutions to problems and solutions may be implemented by both (a friend may lend the other money, etc.); in a therapeutic relationship, solutions are discussed but are only implemented by the patient. E-In a social relationship, communication is usually deep and evaluated; in a therapeutic relationship, communication remains on a more superficial level, allowing patients to feel comfortable.

A-In a social relationship, both parties' needs are met; in a therapeutic relationship, only the patient's needs are to be considered. C-Giving advice is done in social relationships; in therapeutic relationships giving advice is not usually therapeutic. D-In a social relationship, both parties come up with solutions to problems and solutions may be implemented by both (a friend may lend the other money, etc.); in a therapeutic relationship, solutions are discussed but are only implemented by the patient.

The nurse is working with a client experiencing depression stemming from low self-esteem. The client is distrustful of unit staff and "just wants to go home." Initially what is the nurse's priority? A-Making the client feel physically and emotionally safe B-Teaching the client effective coping skills C-Identifying the client's positive traits D-Focusing on preparing the client for a speedy discharge

A-Making the client feel physically and emotionally safe Maslow describes safety as a basic need, meaning that it is so basic to existence that it must be resolved to reduce the tension associated with it. These needs have the greatest strength and must be satisfied before a person turns his attention to higher level needs

A nurse spends extra time with a client who has personality features similar to the nurse's estranged spouse. Which aspect of countertransference is most likely to result? A-Overinvolvement B-Misuse of honesty C-Indifference D-Rescue

A-Overinvolvement

Which nursing intervention demonstrates the theory behind operant conditioning? A-Rewarding the client with a token for avoiding an argument with another client B-Showing the client how to be assertive without being aggressive C-Demonstrating deep breathing techniques to a group of clients D-Explaining to the client the consequences of not following unit rules

A-Rewarding the client with a token for avoiding an argument with another client Operant conditioning is the basis for behavior modification and uses positive reinforcement to increase desired behaviors. For example, when desired goals are achieved or behaviors are performed, patients might be rewarded with tokens. These tokens can be exchanged for food, small luxuries, or privileges. This reward system is known as a token economy. None of the remaining options demonstrate reward for positive behaviors, climate, and structure, for healing.

When discussing therapy options, the nurse should provide information about interpersonal therapy to which patient? Select all that apply. A-The teenager who is the focus of bullying at school B-The older woman who has just lost her life partner to cancer C-The young adult who has begun demonstrating hoarding tendencies D-The adolescent demonstrating aggressive verbal and physical tendencies E-The middle-age adult who recently discovered her partner has been unfaithful

A-The teenager who is the focus of bullying at school B-The older woman who has just lost her life partner to cancer E-The middle-age adult who recently discovered her partner has been unfaithful

A male patient reports to the nurse "I'm told I have memories of childhood abuse stored in my unconscious mind. I want to work on this." Based on this statement, what information should the nurse provide to the patient? A-To seek the help of a trained therapist to help uncover and deal with the trauma associated with those memories B-how to use a defense mechanism such as suppression so that the memories will be less threatening. C-Psychodynamic therapy will allow the surfacing of those unconscious memories to occur in just a few sessions D-Group sessions are valuable to identifying underlying themes of the memories being suppressed

A-To seek the help of a trained therapist to help uncover and deal with the trauma associated with those memories

A client preparing for discharge presents the nurse with a handmade card of appreciation for care this nurse provided. Should the nurse accept the card? A-Yes B-No C-Not Sure

A-Yes

A client states "That nurse never seems comfortable being with me." The nurse can be described as A-not seeming genuine to the client. B-transmitting fear of clients. C-unfriendly and aloof. D-controlling.

A-not seeming genuine to the client. Hiding behind a role, using stiff or formal interactions, and creating distance between self and client suggest a nurse is lacking in genuineness, or the ability to interact in a person-to-person fashion.

According to Maslow's heirarchy of needs, the most basic needs category for nurses to address are: A-physiological B-safety C-love and belonging D-self-actualization

A-physiological

In an outpatient psychiatric clinic, a nurse notices that a newly admitted young patient smiles when he sees her. One day the young man tells the nurse, "You are pretty like my mother." The nurse recognizes that the patient is exhibiting: A-transference B-Id expression C-Countertransference D-A cognitive distortion

A-transference

Which question should the nurse ask when assessing for what Sullivan's Interpersonal Theory identifies as the most painful human condition? A-"Is self-esteem important to you?" B-"Do you think of yourself as being lonely?" C-"What do you do to manage your anxiety?" D-"Have you ever been diagnosed with depression?"

B-"Do you think of yourself as being lonely?"

Consider both Sullivan's term security operations and Freud's term defense mechanisms. Which statement suggests that the client's specialized treatment goal has been successfully met? A-"I really think I can succeed in school now." B-"I'm experiencing much less anxiety about school now." C-"Going back to school is hard and I'll need support." D-"I know that I'm not the only person who has a difficult time in school."

B-"I'm experiencing much less anxiety about school now." Both Sullivan and Freud coined terms to mean actions that individuals do that are an attempt to reduce anxiety. The terms to do not refer to activities that increase self-esteem. Security operations and defense mechanisms are not conscious and therefore do not increase self-awareness.

A nurse seeks to establish a relationship with a patient readmitted to the hospital. The patient has bipolar disorder, depressed type, and was hospitalized the preceding month. Which statement by the nurse would contribute to establishing trust? A-"Weren't you complying with your medication regimen?" B-"It must be discouraging to be readmitted to the hospital so soon" C-Everyone with bipolar disorder ends up in the hospital occasionally" D-"You must take your drugs and prescribed or you will be rehospitalized"

B-"It must be discouraging to be readmitted to the hospital so soon"

A nurse on the psychiatric unit has a past history of alcoholism and has regular meetings with a mentor. Which statement made to the nurse's mentor would indicate the presence of countertransference? A-"My patient is being discharged tomorrow. I provided discharge teaching and stressed the importance of calling the help line number should she become suicidal again." B-"My patient has been abusing alcohol. I told her that the only way to recover was to go 'cold turkey' and to get away from her dysfunctional family and to do it now!" C-"My patient started drinking after 14 years of sobriety. We are focusing on his treatment plan of attending AA (Alcoholics Anonymous) meetings five times a week after discharge." D-"My patient, is an elderly woman with depression. She calls me by her daughter's name because she says I remind her of her daughter."

B-"My patient has been abusing alcohol. I told her that the only way to recover was to go 'cold turkey' and to get away from her dysfunctional family and to do it now!" This statement indicates countertransference; the nurse may be overidentifying with the patient because of a past history of alcoholism. Providing adamant advice to the patient that, besides being nontherapeutic, may be more relevant to personal past experiences than to the patient's. The discharge teaching for a patient being discharged and focusing on the treatment plan for the alcoholic patient are appropriate and show no signs of countertransference. The patient calling the nurse by her daughter's name is transference rather than countertransference.

Using Maslow's model of needs, the nurse providing care for an anxious client identifies which intervention as being a priority? A-Assessing the client's ability to fulfill appropriate developmental level tasks B-Assessing the client for strengths upon which a nurse-client relationship can be based C-Planning one-on-one time to assist in identifying the fears trigger the client's anxiety D-Evaluating the client's ability to learn and retain essential information regarding their current condition

B-Assessing the client for strengths upon which a nurse-client relationship can be based The value of Maslow's model in nursing practice is twofold. First, the emphasis on human potential and the client's strengths is key to successful nurse-client relationships. The second value lies in establishing what is most important in sequencing of nursing actions in the nurse-client relationship.

A client tells the nurse, "I have something secret to tell you, but you can't tell anyone else." The nurse agrees. What is the likely consequence of the nurse's action? A-Healthy feelings of sympathy by the nurse towards the client B-Blurred boundaries in the nurse-client relationship C-Improved rapport between the nurse and client D-Enhanced trust between the nurse and client

B-Blurred boundaries in the nurse-client relationship

Freud believed that individuals cope with anxiety by implementing which mechanism? A-The superego B-Defense mechanisms C-Security operations D-Cognitive distortions

B-Defense mechanisms The ego develops defenses or defense mechanisms to ward off anxiety by preventing conscious awareness of threatening feelings. None of the other options were proposed by Freud as a mechanism for dealing with anxiety.

The nurse is caring for an adult client who experienced severe physical abuse from the age of 2 through 12. What information should the nurse provide the client concerning the function of the "id" and the ability to function as an adult? A-It has control over the emotional frustration felt as an adult. B-It is the source of one's survival instincts. C-It is severely damaged by abuse experienced before the age of 5 years. D-It provides an individual with the ability to differentiate believed and real experiences.

B-It is the source of one's survival instincts. Freud delineated three major and distinct but interactive systems of the human personality. At birth we are all id. The id is the source of all drives, instincts, reflexes, needs, genetic inheritance, and capacity to respond as well as all the wishes that motivate us. The id provides an individual with the instincts to survive the emotional trauma associated with physical abuse.

Role-playing is associated with which type of psychotherapy? A-Psychoanalysis B-Modeling C-Operant conditioning D-Systematic desensitization

B-Modeling

During what stage of the therapeutic nurse-client relationship is a formal or informal contract between the nurse and client established? A-Preorientation B-Orientation C-Working D-Termination

B-Orientation Contracting is part of the orientation phase of the relationship. Establishing the operational "rules" provides a foundation for the relationship.

A client states, "I will always be alone because nobody could ever love me." The nurse recognizes that the client is expressing what cognitive-behavioral concept? A-Emotional consequence B-Schema C-Actualization D-Aversion

B-Schema Schemas are unique assumptions about ourselves, according to Beck's theory. This statement is an example of a negative schema. Emotional consequence is the end result of negative thinking process, as described by Ellis. Actualization is a level of Maslow's Hierarchy of Needs. Aversion is a therapy characterized by punishment.

Which theorist is associated with behavioral therapy? A-Freud B-Skinner C-Sullivan D-Peplau

B-Skinner B.F. Skinner (1904-1990) represented the second wave of behavioral theorists and is recognized as one of the prime movers behind the behavioral movement.

When a nurse and client meet informally or have an otherwise limited but helpful relationship, what term is used to identify this relationship? A-Crisis intervention B-Therapeutic encounter C-Autonomous interaction D-Preorientation phenomenon

B-Therapeutic encounter A therapeutic encounter is a short but helpful interaction between the nurse and client.

Schemata, automatic thoughts, and cognitive distortions are terms that relate to A-rational-emotive behavioral therapy B-cognitive-behavioral therapy C-operant conditioning therapy D-biofeedback

B-cognitive-behavioral therapy Rapid, unthinking responses based on schemas are known as automatic thoughts. Automatic thoughts, or cognitive distortions, are often irrational and lead to false interpretations and cognitive errors. Making changes to these cognitive errors is part of cognitive-behavioral therapy.

Which theorist most influenced the professional practice of psychiatric nursing? A-Harry Stack Sullivan B-Hildegard Peplau C-Erik Erikson D-Ivan Pavlov

B.Hildegard Peplau Peplau not only established the foundation for the professional practice of psychiatric nursing, she also continued to enrich psychiatric nursing theory and work for the advancement of nursing practice throughout her career.

A patient is telling a tearful story. The nurse listens empathically and responds therapeutically with: A-"The next time you find yourself in a similar situation, please call me" B-"I am sorry this situation made you feel so badly. Would like some tea?" C-"Let's devise a plan on how you will react the next time you are in a similar situation." D-"I am sorry that your friend was so thoughtless. You should be treated better."

C-"Let's devise a plan on how you will react the next time you are in a similar situation."

A patient who recently loss a parent begins crying during a one-to-one session with the nurse. Which of the following responses by the nurse illustrates empathy? A-"I'm so sorry. My father died 2 years ago, so I know how you are feeling." B-"You need to focus on yourself right now. You deserve to take time just for you." C-"That must have been such a hard situation for you to deal with." D-"I know that you will get over this. It just takes time."

C-"That must have been such a hard situation for you to deal with." This response reflects understanding of the patient's feelings, which is empathy. Feeling sorry for the client represents sympathy, whereas not addressing the patient's concern belittles the patient's feelings of grief she is expressing by changing the subject. Telling the patient she will get over it does not reflect empathy and is closed-ended.

How does Harry Stack Sullivan's Interpersonal Theory view anxiety? A-An emotional experience felt after the age of 5 years. B-A sign of guilt in adults. C-A painful emotion arising from social insecurity. D-The result of trying to go beyond experiences of guilt and pain.

C-A painful emotion arising from social insecurity. According to Sullivan, the purpose of all behavior is to get needs met through interpersonal interactions and decrease or avoid anxiety. He viewed anxiety as a key concept and defined it as any painful feeling or emotion arising from social insecurity or blocks to getting biological needs satisfied.

According to Freud, a client experiencing dysfunction of the conscious as part of the mind will have problems with which aspect of memory? A-Recent memory B-Long-term memory C-All memories D-Painful memories

C-All memories Freud described the conscious part of the mind as containing all of the material that the person is aware of at any one time and so as dysfunction of the conscious mind involves all memories

A term is a synonym for the characteristic of genuineness? A-Respect B-Empathy C-Authentic D-Positive regard

C-Authentic Genuineness refers the nurse's ability to be open, honest, and authentic in interactions with patients. It is the ability to meet others person-to-person without hiding behind roles.

The nurse is planning care for a 14-year-old. The nurse demonstrates an understanding of the developmental task appropriate for this client by providing which experience? A-Spending one-on-one time with staff to establish trust B-Providing them with the opportunity to select which unit activities they will participate in to gain autonomy C-Encouraging them to talk about their school plans to help achieve identity D-Assign them to help clean up the dayroom to develop a sense of industry

C-Encouraging them to talk about their school plans to help achieve identity According to Erikson, the task of adolescence is to achieve identity rather than to be left in role confusion. A sense of identity is essential to making the transition into adulthood. While appropriate activities none of the options are specifically identified with the developmental task for a 14-year-old.

A nurse is providing care to a 28-year-old patient diagnosed with bipolar disorder who was admitted in a manic state. According to Maslow's Hierarchy of Needs theory, the nurse should identify which patient symptom as having priority? A-Rapid, pressured speech B-Grandiose thoughts C-Lack of sleep D-Hyperactive behavior

C-Lack of sleep Based on Maslow's theory, physiological needs such as food, water, air, sleep, etc., are the priority and must be taken care of first. The other options are symptoms of mania but not as critical as lack of sleep

When considering the suggestions of Hildegard Peplau, which activity should the nurse regularly engage in to ensure that the patient stays the focus of therapeutic conversations? A-Assessing the patient for unexpressed concerns and fears B-Evaluating the possible need for additional training and education C-Reflecting on personal behavior and personal needs D-Avoiding power struggles with a manipulative patient

C-Reflecting on personal behavior and personal needs

According to Freud, which aspect of the personality motivates the individual to seek perfection? A- Id B- Ego C-Superego D-Not sure

C-Superego The superego represents the ideal rather than real; it seeks perfection as opposed to seeking pleasure or engaging reason

What is the primary difference between a social and a therapeutic relationship? A-Type of information exchanged B-Amount of satisfaction felt C-Type of responsibility involved D-Amount of emotion invested

C-Type of responsibility involved In a therapeutic relationship the nurse assumes responsibility for focusing the relationship on the client's needs, facilitating communication, assisting the client with problem-solving, and helping the client identify and test alternative coping strategies.

A patient is admitted to your unit who has an uncanny resemblance to your older sister. As a child, your older sister bossed you around and criticized you constantly. You realize that you are responding negatively to this patient. What is going on and what should the nurse do?

Countertransferrance - unconscious feelings the health care worker is having towards the patient. The nurse should realize the importance of maintaining self-awareness and seeking supervisory guidance ans the therapeutic relationship progresses

Consider the nurse-patient relationship on an inpatient psychiatric unit. Which of the following statements made by the nurse reflects an accurate understanding of when the issue of termination should first be discussed? A-"You are being discharged today, so I'd like to bring up the subject of termination—discussing your time here and summarizing what coping skills you have attained." B-"I haven't met my new patient yet, but I am working through my feelings of anxiety in dealing with a patient who wanted to kill herself." C-"Now that we are working on your problem-solving skills and behaviors you'd like to change, I'd like to bring up the issue of termination." D-"Now that we've discussed your reasons for being here and how often we will meet, I'd like to talk about what we will do at the time of your discharge."

D-"Now that we've discussed your reasons for being here and how often we will meet, I'd like to talk about what we will do at the time of your discharge." The issue of termination is brought up first in the orientation phase.

A client has been admitted to your inpatient psychiatric unit with suicidal ideation. In a one-to-one session with the nurse, he shares the terrible guilt he feels over sexually abusing his stepdaughter and wanting to die because of it. Which of the following responses you could make reflects a helpful trait in a therapeutic relationship? A-"It's good that you feel guilty. That means you still have a chance of being helped." B-"Of course you feel guilty. You did a horrendous thing. You shouldn't even forget what you did." C-"The biggest question is, will you do it again? You will end up having even worse guilt feelings because you hurt someone again." D-"You are suffering with guilt over what you did. Let's talk about some goals we could work on that may make you want to keep living."

D-"You are suffering with guilt over what you did. Let's talk about some goals we could work on that may make you want to keep living." This response demonstrates suspending value judgment, a helpful trait in establishing and maintaining a therapeutic relationship. Although it is difficult, nurses are more effective when they don't use their own value systems to judge patients' thoughts, feelings, or behaviors. The other options are all judgmental responses. Judgment on the part of the nurse will most likely interfere with further explorations of feelings and hinder the therapeutic relationship

Which action reflects therapeutic practices associated with operant conditioning? A-Encouraging a patient to read to their children to foster a love for learning B-Encouraging a patient to make daily journal entries describing their feelings C-Suggesting to a new mother that she spend time cuddling her newborn during the day D-Acknowledging a patient who is often verbally aggressive for complimenting a picture another patient drew

D-Acknowledging a patient who is often verbally aggressive for complimenting a picture another patient drew

A nurse is assessing a patient who graduated at the top of his class but now obsesses about being incompetent in his new job. The nurse recognizes that this patient may benefit from which of the following type of psychotherapy: A-Interpersonal B-Operant Conditioning C-Behavioral D-Cognitive-Behavioral

D-Cognitive-Behavioral

The outcome of the nurse's expressions of sympathy instead of empathy toward the client often leads to which outcome? A-Enhanced client coping B-Lessening of client emotional pain C-Increased hope for client improvement D-Decreased client communication

D-Decreased client communication Sympathy and the resulting projection of the nurse's feelings limit the client's opportunity to further discuss the problem.

Which client problem would be most suited to the use of interpersonal therapy? A-Disturbed sensory perception B-Impaired sensory perception C-Medication noncompliance D-Dysfunctional grieving

D-Dysfunctional grieving Interpersonal therapy is considered to be effective in resolving problems of grief, role disputes, role transition, and interpersonal deficit

The nurse providing anticipatory operant conditioning guidance to the mother of a toddler should advise that childhood temper tantrums are best handled by which intervention? A-Giving the child what he/she is asking for B-Scolding the child when he/she displays tantrum behaviors C-Spanking the child at the onset of the tantrum behaviors D-Ignoring the tantrum and giving attention when the child acts appropriately

D-Ignoring the tantrum and giving attention when the child acts appropriately Ignoring the tantrum provides no reinforcement of the undesirable behavior. Instead, approval and reinforcement are given when the child is behaving in the desired way. This is an example of absence of reinforcement, or extinction.

A nurse expresses an exclusive belief in the biological model for mental illness when stating "it's the only one I really believe." What conclusion should be drawn from this statement? A-The biological model is the oldest and most reliable model for explaining mental illness. B-The biological model has been proven to be successful in finding the cause of most symptoms of mental illness. C-The biological model is the most popular theory among leading psychiatrists and therefore the one that should be fully embraced. D-In believing only in the biological model, other influences on mental health including cultural, environmental, social, and spiritual influences are not taken into account.

D-In believing only in the biological model, other influences on mental health including cultural, environmental, social, and spiritual influences are not taken into account. In believing only in the biological model to the exclusion of other theories and perspectives, influences such as educational, social, spiritual, cultural, environmental, and economic are not considered, and these have also been proven to play a part in mental health and mental illness.

A client tells the nurse "I really feel close to you. You are like the friend I never had." The nurse can assess this statement as indicating the client may be experiencing which unconscious emotion? A-Congruence B-Empathetic feelings C-Countertransference D-Positive transference

D-Positive transference Transference involves the client experiencing feelings toward a nurse that belong to a significant person in the client's past. If a patient is motivated to work with you, completes assignments between sessions, and shares feelings openly, it is likely the patient is experiencing positive transference.

Which phase of the nurse-client relationship may cause client anxieties to reappear and past losses to be reviewed? A-Preorientation phase B-Orientation phase C-Working phase D-Termination phase

D-Termination phase Termination, a stage in which the client must face the loss or ending of the therapeutic relationship, often reawakens the pain of earlier losses.

A child with many "risk factors" for the development of mental illness develops. You hear the term "resilience" used. What are the characteristics this child has?

The resilient child has the following characteristics: 1-adaptability to changes in environment 2-ability to form nurturing relationships with other adults 3-ability to distance self from emotional chaos 4-good social intelligence 5-good problem-solving skills 6-ability to perceive long-term future

Which statement demonstrates a well-structured attempt at limit setting? a. "Hitting me when you are angry is unacceptable." b. "I expect you to behave yourself during dinner." c. "Come here, right now!" d. "Good boys don't bite."

a. "Hitting me when you are angry is unacceptable."

Mary is a 39-year-old attending a psychiatric outpatient clinic. Mary believes that her husband, sister, and son cause her problems. Listening to Mary describe the problems the nurse displays therapeutic communication in which response? a. "I understand you are in a difficult situation." b. "Thinking about being wronged repeatedly does more harm than good." c. "I feel bad about your situation, and I am so sorry it is happening to you and your family." d. "It must be so difficult to live with uncaring people."

a. "I understand you are in a difficult situation."

The patient expresses sadness at "being all alone with no one to share my life with." Which response by the nurse demonstrates the existence of a therapeutic relationship? a. "Loneliness can be a very painful and difficult emotion." b. "Let's talk and see if you and I have any interests in common." c. "I use Facebook to find people who share my love of cooking." d. "Loneliness is managed by getting involved with people."

a. "Loneliness can be a very painful and difficult emotion."

Which patient statement demonstrates a value held regarding children? a. "Nothing is more important to me than the safety of my children." b. "I believe my spouse wants to leave both me and our children." c. "I don't think my child's success depends on going to college." d. "I know my children will help me through my hard times."

a. "Nothing is more important to me than the safety of my children."

Carolina is surprised when her patient does not show for a regularly scheduled appointment. When contacted, the patient states, "I don't need to come see you anymore. I have found a therapy app on my phone that I love." How should Carolina respond to this news? a. "That sounds exciting, would you be willing to visit and show me the app?" b. "At this time, there is no real evidence that the app can replace our therapy." c. "I am not sure that is a good idea right now, we are so close to progress." d. "Why would you think that is a better option than meeting with me?"

a. "That sounds exciting, would you be willing to visit and show me the app?"

During an admission assessment and interview, which channels of information communication should the nurse be monitoring? Select all that apply. a. Auditory b. Visual c. Written d. Tactile e. Olfactory

a. Auditory b. Visual d. Tactile e. Olfactory

When should a nurse be most alert to the possibility of communication errors resulting in harm to the patient? a. Change of shift report b. Admission interviews c. One-to-one conversations with patients d. Conversations with patient families

a. Change of shift report

What principle about nurse-patient communication should guide a nurse's fear about "saying the wrong thing" to a patient? a. Patients tend to appreciate a well-meaning person who conveys genuine acceptance, respect, and concern for their situation. b. The patient is more interested in talking to you than listening to what you have to say and so is not likely to be offended. c. Considering the patient's history, there is little chance that the comment will do any actual harm. d. Most people with a mentally illness have by necessity developed a high tolerance of forgiveness.

a. Patients tend to appreciate a well-meaning person who conveys genuine acceptance, respect, and concern for their situation.

Which of the following statements represent a nontherapeutic communication technique? Select all that apply. a."Why didn't you attend group this morning?" b."From what you have said, you have great difficulty sleeping at night." c."What did your boyfriend do that made you leave? Are you angry at him? Did he abuse you in some way?" d."If I were you, I would quit the stressful job and find something else." e."I'm really proud of you for the way you stood up to your brother when he visited today." f."You mentioned that you have never had friends. Tell me more about that." g."It sounds like you have been having a very hard time at home lately."

a."Why didn't you attend group this morning?" c."What did your boyfriend do that made you leave? Are you angry at him? Did he abuse you in some way?" d."If I were you, I would quit the stressful job and find something else." e."I'm really proud of you for the way you stood up to your brother when he visited today." All these options reflect the nontherapeutic techniques of (in order) asking "why" questions; using excessive questioning; giving advice; and giving approval. The other options describe therapeutic techniques of restating, exploring, and reflecting.

Which child would be most difficult to diagnose a neurodevelopmental disorder? a.3 year old b.5 year old c.8 year old d.12 year old

a.3 year old

When preparing to hold an admission interview with a client, the nurse pulls up a chair and sits facing the client with his or her knees almost touching. When the nurse leans in close to speak, the client becomes visibly flustered and gets up and leaves the room. What is the most likely explanation for client's behavior? a.The nurse violated the client's personal space by physically being too close. b.The client has issues with sharing personal information. c.The nurse failed to explain the purpose of the admission interview. d.The client is responding to the voices by ending the conversation.

a.The nurse violated the client's personal space by physically being too close. By sitting and leaning in so closely, the nurse has entered into intimate space (0 to 18 inches), rather than social distance. This has likely made the patient may feel uncomfortable with being so close to someone unknown to them

What is the focus during clinical supervision? a.The nurse's behavior in the nurse-client relationship b.Analysis of the client's motivation for transferences c.Devising alternative strategies for client growth d.Assisting the client to develop increased independence

a.The nurse's behavior in the nurse-client relationship Clinical supervision helps the nurse look at his or her own behavior and determine more effective approaches to working with clients.

Which statement demonstrates that a parent understands the diagnosis of attention deficit hyperactivity disorder? a."My child will never be able to graduate or go to college but may be able to learn a vocational skill." b-"My child's performance will improve in a structured setting that provides rewards for appropriate behavior." c. "Noting is wrong with my child. The school hasn't provided qualified teachers and classroom settings." d."My child is just going through a stage this problem will go away with time."

b-"My child's performance will improve in a structured setting that provides rewards for appropriate behavior."

Which statement made by the nurse demonstrates the best understanding of nonverbal communication? a. "The patient's verbal and nonverbal communication is often different." b. "When my patient responds to my question, I check for congruence between verbal and nonverbal communication to help validate the response." c. "If a patient is slumped in the chair, I can be sure he's angry or depressed." d. "It's easier to understand verbal communication that nonverbal communication."

b. "When my patient responds to my question, I check for congruence between verbal and nonverbal communication to help validate the response."

Which statement made by either the nurse or the patient demonstrates an ineffective patient-nurse relationship? a. "I've given a lot of thought about what triggers me to be so angry." b. "Why do you think it's acceptable for you to be so disrespectful to staff?" c. "Will your spouse be available to attend tomorrow's family group session?" d. "I wanted you to know that the medication seems to be helping me fell less anxious."

b. "Why do you think it's acceptable for you to be so disrespectful to staff?"

You have been working closely with a patient for the past month. Today he tells you he is looking forward to meeting with his new psychiatrist but frowns and avoids eye contact while reporting this to you. Which of the following responses would most likely be therapeutic? a. "A new psychiatrist is a chance to start fresh; I'm sure it will go well for you." b. "You say you look forward to the meeting, but you appear anxious or unhappy." c. "I notice that you frowned and avoided eye contact just now. Don't you feel well?" d. "I get the impression you don't really want to see your psychiatrist—can you tell me why?"

b. "You say you look forward to the meeting, but you appear anxious or unhappy."

Which patient outcome is directly associated with the goals of a therapeutic nurse-patient relationship? a. Patient will be respectful of other patients on the unit. b. Patient will identify suicidal feelings to staff whenever they occur. c. Patient will engage in at least one social interaction with the unit population daily. d. Patient will consume a daily diet to meet both nutritional and hydration needs.

b. Patient will identify suicidal feelings to staff whenever they occur.

A male patient frequently inquires about the female student nurse's boyfriend, social activities, and school experiences. Which is the best initial response by the student? a. The student requests assignment to a patient of the same gender as the student. b. She limits sharing personal information and stresses the patient-centered focus of the conversation. c. The student shares information to make the therapeutic relationship more equal. d. She explains that if he persists in focusing on her, she cannot work with him.

b. She limits sharing personal information and stresses the patient-centered focus of the conversation.

Which student behavior is consistent with therapeutic communication? a. Offering your opinion when asked to convey support. b. Summarizing the essence of the patient's comments in your own words. c. Interrupting periods of silence before they become awkward for the patient. d. Telling the patient he did well when you approve of his statements or actions.

b. Summarizing the essence of the patient's comments in your own words.

What is the most helpful nursing response to a client who reports thinking of dropping out of college because it is too stressful? a."Don't let them beat you! Fight back!" b."School is stressful. What do you find most stressful?" c."I know just what you are going through. The stress is terrible." d."You have only two more semesters. You will be glad if you stick it out."

b."School is stressful. What do you find most stressful?" This response acknowledges the speaker's perception of school as difficult and asks for further information. This response suggests the nurse is listening actively and is concerned

The preferred seating arrangement for a nurse-client interview should incorporate which positioning? a.The nurse behind a desk and the client in a chair in front of the desk. b.The nurse and client sitting at a 90-degree angle to each other. c.The client sitting in a chair and the nurse standing a few feet away. d.The nurse and client sitting facing each other.

b.The nurse and client sitting at a 90-degree angle to each other This arrangement allows the nurse to observe the client but places no barriers between the principals. The two are at the same height, so neither is in an inferior position. Face-to-face seating is a more confrontational arrangement and therefore more anxiety producing

A 55-year-old patient recently came to the United States from England on a work visa. The patient was admitted for severe depression following the death of a life partner weeks ago. While discussing the death and its effects the patient shows little emotion. Which of the following explanations is most plausible for this lack of emotion? a.The patient in denial. b.The response may reflect cultural norms. c.The response may reflect personal guilt. d.The patient may have an antisocial personality.

b.The response may reflect cultural norms. Showing little emotion while in distress may be a cultural phenomenon. Some cultures, such as the British and German cultures, tend to value highly the concept of self-control and may show little facial emotion in the presence of emotional turmoil. There is no evidence to suggest the patient's lack of emotion is a result of any of the other options.

What therapeutic communication technique is the nurse using by asking a newly admitted patient, "Please tell me what was happening that led to your hospitalization here?" a.Using a minimal encourager b.Using an open-ended question c.Paraphrasing d.Reflecting

b.Using an open-ended question Open-ended questions require more than one-word answers. This question encourages the patient to provide a narrative concerning the circumstances surrounding the need for admissio

A 4-year-old frequently lashes out in anger at adults and other children. The child's style of behavior is an aspect of a.neurobiology b.temperment c.resilience d. culture

b.temperment

Morgan is a third-year nursing student in her psychiatric clinical rotation. She is assigned to an 80-year-old widow admitted for major depressive disorder. The patient describes many losses and sadness. Morgan becomes teary and says meaningfully, "I am so sorry for you." Morgan's instructor overhears the conversation and says, "I understand that getting tearful is a human response. Yet, sympathy isn't helpful in this field." The instructor urges Morgan to focus on: a. "Adopting the patient's sorrow as your own." b. "Maintaining pure objectivity."' c. "Using empathy to demonstrate respect and validation of the patient's feelings." d. "Using touch to let her know that everything is going to be alright."

c. "Using empathy to demonstrate respect and validation of the patient's feelings."

James is a 42-year-old patient with schizophrenia. He approaches you as you arrive for day shift and anxiously reports, "Last night, demons came to my room and tried to rape me." Which response would be most therapeutic? a. "There are no such things as demons. What you saw were hallucinations." b. "It is not possible for anyone to enter your room at night. You are safe here." c. "You seem very upset. Please tell me more about what you experienced last night." d. "That must have been very frightening, but we'll check on you at night and you'll be safe."

c. "You seem very upset. Please tell me more about what you experienced last night."

Emily is a 28-year-old nurse who works on a psychiatric unit. She is assigned to work with Jenna, a 27-year-old who was admitted with major depressive disorder. Emily and Jenna realize that they graduated from the same high school and each has a 2-year-old daughter. Emily and Jenna discuss getting together for a play date with their daughters after Jenna is discharged. This situation reflects: a. Successful termination b. Promoting interdependence c. Boundary blurring d. A strong therapeutic relationship

c. Boundary blurring

A registered nurse is caring for an older male who reports depressive symptoms since his wife of 54 years died suddenly. He cries, maintains closed body posture, and avoids eye contact. Which nursing action describes attending behavior? a. Reminding the patient gently that he will "feel better over time" b. Using a soft tone of voice for questioning c. Sitting with the patient and taking cues for when to talk or when to remain silent d. Offering medication and bereavement services

c. Sitting with the patient and taking cues for when to talk or when to remain silent

With which client should the nurse make the assessment that not using touch would probably be in the client's best interests? a.A recent immigrant from Russia b.A deeply depressed client c.A Chinese American client d.A tearful client reporting pain

c.A Chinese American client Chinese Americans may not like to be touched by strangers since it is a cultural characteristic.

Of the following environments, which would be most conducive to a therapeutic session? a.The nurses' station b.A table in the coffee shop c.A quiet section of the day room d.The utility room

c.A quiet section of the day room Of the options provided, a quiet corner of the day room offers the safest, quietest, most private environment for a therapeutic encounter

A patient is sitting with arms crossed over his or her chest, his or her left leg is rapidly moving up and down, and there is an angry expression on his or her face. When approached by the nurse, the patient states harshly, "I'm fine! Everything's great." Which statement related to communication should the nurse focus on when working with this patient? a.Verbal communication is always more accurate than nonverbal communication. b.Verbal communication is more straightforward, whereas nonverbal communication does not portray what a person is thinking. c.Nonverbal and verbal communication may be different; nurses must pay attention to the nonverbal communication being presented to get an accurate message. d.Nonverbal communication is about 10% of all communication, and verbal communication is about 90%.

c.Nonverbal and verbal communication may be different; nurses must pay attention to the nonverbal communication being presented to get an accurate message. Communication is roughly 10% verbal and 90% nonverbal, so nurses must pay close attention to nonverbal cues to accurately assess what the patient is really feeling. The other options are all untrue of verbal and nonverbal communication and are actually the opposite of what is believed of communication

The client makes the decision to sit about 5 feet away from the nurse during the assessment interview. The nurse can accurately make what assumption about the client's perception of the nurse? a.The nurse is a safe person to interact with. b.The nurse is a new friend. c.They view the nurse as a stranger. d.They view the nurse as a peer.

c.They view the nurse as a stranger. Social distance (4-12 feet) is reserved for strangers or acquaintances. This is often the client's perception of staff during the initial phase of relationship-building.

Which nursing statement is an example of reflection? a. "I think this feeling will pass." b. "So you are saying that life has no meaning." c. "I'm not sure I understand what you mean." d. "You look sad."

d. "You look sad."

Therapeutic communication is the foundation of a patient- centered interview. Which of the following techniques is not considered therapeutic? a. Restating b. Encouraging description of perception c. Summarizing d. Asking "why" questions

d. Asking "why" questions

What is the greatest trigger for the development of a patient's nurse focused transference? a. The similarity between the nurse and someone the patient already dislikes b. The nature of the patient's diagnosed mental illness c. The history the patient has with their parents d. The degree of authority the nurse has over the patient

d. The degree of authority the nurse has over the patient

A patient is presenting with behaviors that indicate anger. When approached, the patient states harshly, "I'm fine! Everything's great." Which response should the nurse provide to the patient? a."Okay, but we are all here to help you, so come get one of the staff if you need to talk." b."I'm glad everything is good. I am going to give you your schedule for the day and we can discuss how the groups are going." c."I don't believe you. You are not being truthful with me." d."It looks as though you are saying one thing but feeling another. Can you tell me what may be upsetting you?"

d."It looks as though you are saying one thing but feeling another. Can you tell me what may be upsetting you?" This response uses the therapeutic technique of clarifying; it addresses the difference between the patient's verbal and nonverbal communication and encourages sharing of feelings. The other options do not address the patient's obvious distress or are confrontational and judgmental.

During a clinical interview the client falls silent after disclosing that she was sexually abused as a child. The nurse should engage in which intervention in response to the client's silence? a.Quickly break the silence and encourage the client to continue. b.Reassure the client that the abuse was not her fault. c.Reach out and gently touch the client's arm. d.Allow the client to break the silence.

d.Allow the client to break the silence. Silence is not a "bad" thing. It gives the speaker time to think through a point or collect his or her thoughts

Which communication techniques should the nurse use with a client who has been identified as having difficulty expressing thoughts and feelings? a.Using emotionally charged words and gestures b.Offering opinions and avoiding periods of silence c.Asking closed-ended questions requiring "yes" or "no" answers d.Asking open-ended questions and seeking clarification

d.Asking open-ended questions and seeking clarification Open-ended questions give the client the widest possible latitude in answering. Also, the client can take the lead in the interview. Seeking clarification helps the client clarify his or her own thoughts and promotes mutual understanding.

When discussing her husband, a client shares that "I would be better off alone. At least I would be able to come and go as I please and not have to be interrogated all the time." What therapeutic communication technique is the nurse using when responding, "Are you saying that things would be better if you left your husband?" a.Focusing b.Restating c.Reflection d.Clarification

d.Clarification Clarification verifies the nurse's interpretation of the client's message. None of the other options are associated with the verification of the client's meaning.

After a client discusses his/her relationship with his/her father, the nurse asks, "Tell me if I'm correct that you feel dominated and controlled by him?" What is the purpose of the nurse's question? a.Eliciting more information b.Encouraging evaluation c.Verbalizing the implied d.Clarifying the message

d.Clarifying the message Clarification helps the nurse understand and correctly interpret the client's message. It gives the client the opportunity to correct misconceptions. This is not the purpose of any of the other options.

During a therapeutic encounter, the nurse makes an effort to ensure the use of two congruent levels of communication. What is the rationale for this? a.The mental image of a word may not be the same for both nurse and client. b.One statement may simultaneously convey conflicting messages. c.Many of the client's remarks are no more than social phrases. d.Content of messages may be contradicted by process.

d.Content of messages may be contradicted by process. Verbal messages may be contradicted by the nonverbal message that is conveyed. The nonverbal message is usually more consistent with the client's feelings than the verbal message.

A recent immigrant to the United States from which country would find direct eye contact a positive therapeutic technique? a.Korea b.Mexico c.Japan d.Germany

d.Germany Eye contact conveys interest to most northern European individuals. Eye contact would be considered intrusive to the others.

During a therapeutic encounter the nurse remarks to a client, "I noticed anger in your voice when you spoke of your father. Tell me about that." What communication techniques is the nurse using? a.Giving information and encouraging evaluation b.Presenting reality and encouraging planning c.Clarifying and suggesting collaboration d.Reflecting and exploring

d.Reflecting and exploring Reflecting conveys the nurse's observations of the client when a sensitive issue is being discussed. Exploring seeks to examine a certain idea more fully.


Set pelajaran terkait

Типы текста: повествование, описание, рассуждение

View Set

Anomatopoeia - What is the Anomatopoeia?

View Set

Consumer Behavior Exam 1: Chapters 8, 9, 10, 11

View Set

Ethics Midterm, Ethics Midterm (2)

View Set

Med Term Ch 1 Define/Const Med Terms

View Set

Avoiding Group Harms- U.S. Research Perspectives

View Set